carte olimpiada_balcanica-2010

90
colecŃia

Upload: concurs-calude

Post on 21-Apr-2015

217 views

Category:

Documents


6 download

TRANSCRIPT

Page 1: Carte Olimpiada_balcanica-2010

c o l e c Ń i a

Page 2: Carte Olimpiada_balcanica-2010

Această lucrare este rezultatul colaborării dintre Editura Paralela 45 şi Societatea Română de ŞtiinŃe Matematice.

Editor: Călin Vlasie Corectură: autorii Tehnoredactare: Carmen Rădulescu Coperta colecŃiei: Andrei Mănescu Machetare & prepress: ART CREATIV Descrierea CIP a Bibliotecii NaŃionale a României

ŞERBĂNESCU, DINU Training Problems for the Junior Balkan Mathematical Olympiads – The Romanian Experience / Dinu Şerbănescu, Mircea Fianu, Ioan Şerdean. - Piteşti : Paralela 45, 2010 ISBN 978-973-47-0957-1

I. Fianu, Mircea II. Şerdean, Ioan 51(075.33)(076) Copyright Editura Paralela 45, 2010

Page 3: Carte Olimpiada_balcanica-2010

DDDD INU INU INU INU ŞŞŞŞERBĂNESCUERBĂNESCUERBĂNESCUERBĂNESCU ,,,, FFFFLORICA LORICA LORICA LORICA BBBBANUANUANUANU ,,,, MMMM IRCEA IRCEA IRCEA IRCEA FFFF IANUIANUIANUIANU ,,,, ŞŞŞŞTEFAN TEFAN TEFAN TEFAN SSSSMMMMĂĂĂĂRRRRĂĂĂĂNDNDNDNDOIUOIUOIUOIU ,,,, MMMMARIUS ARIUS ARIUS ARIUS PPPPERIAERIAERIAERIANNNNUUUU ,,,, IIIIOAN OAN OAN OAN ŞŞŞŞERDEANERDEANERDEANERDEAN ,,,, GGGGABRIEL ABRIEL ABRIEL ABRIEL PPPPOPAOPAOPAOPA

TraTraTraTraining Problems ining Problems ining Problems ining Problems for the Junior Balkan for the Junior Balkan for the Junior Balkan for the Junior Balkan

Matehmatical Olympiads Matehmatical Olympiads Matehmatical Olympiads Matehmatical Olympiads

The Romanian ExperienceThe Romanian ExperienceThe Romanian ExperienceThe Romanian Experience

Page 4: Carte Olimpiada_balcanica-2010
Page 5: Carte Olimpiada_balcanica-2010

FOREWORD

The Junior Balkan Mathematical Olympiad started fourteen years ago as an

alternative of the Balkan Olympiads for Junior High students.

Since, the competition has become very popular not only in South-Eastern

European countries but all over the world. Some teams from Asia and Europe

participate as invited countries to the competition.

But the most important achievement of this competition is in my opinion the

long list of nice problems produced for these kids; usually extremely elementary

but containing deep mathematical ideas. Maybe this is why, in the Balkan

countries, the junior teams usually graw to become in years the senior teams for

the BMO and IMO.

The booklet is devoted to the Romanian problems used for the qualification

tests of the Romanian team in the last decade. One can remark the great number of

authors, some of them being IMO students at the time the problems were given.

It is a great job the authors have done here, collecting all those problems and

editing nice solutions.

Radu Gologan

Page 6: Carte Olimpiada_balcanica-2010
Page 7: Carte Olimpiada_balcanica-2010

PROBLPROBLPROBLPROBLEEEEMMMMSSSS

Page 8: Carte Olimpiada_balcanica-2010
Page 9: Carte Olimpiada_balcanica-2010

9999

Chapter IChapter IChapter IChapter I

ALALALALGEBRAGEBRAGEBRAGEBRA

Problem Problem Problem Problem 1111

Prove that for any real numbers a, b, c such that 0 < a, b, c < 1, the following inequality holds

1)1)(1)(1( <−−−+ cbaabc .

Dinu Şerbănescu, 2002

Problem Problem Problem Problem 2222

Let a, b, c be positive real numbers with abc = 1. Prove that

1 + cabcabcba ++

≥++

63.

Mircea Lascu and Vasile Cârtoaje, 2003

Problem Problem Problem Problem 3333

Find the positive real numbers a, b, c which satisfy the inequalities

4(ab + bc + ca) – 1 ≥ a2 + b

2 + c

2 ≥ 3(a3

+ b3 + c

3).

LaurenŃiu Panaitopol, 2004

Problem Problem Problem Problem 4444

Let a, b, c be positive real numbers such that a + b + c ≥ cba

111++ . Prove that

a + b + c ≥ abc

3.

Cezar Lupu, 2005

Problem Problem Problem Problem 5555

Let a, b, c be positive real numbers such that (a + b)(b + c)(c + a) = 1. Show that

ab + bc + ca ≤ 4

3.

Cezar Lupu, 2005

Problem Problem Problem Problem 6666

Let a, b, c be positive real numbers with a + b + c = 3. Prove that

(3 – 2a)(3 – 2b)(3 – 2c) ≤ a2b

2c

2.

Robert Szasz, 2005

Page 10: Carte Olimpiada_balcanica-2010

10101010

PrPrPrProblem oblem oblem oblem 7777

Prove that cbaba

c

ca

b

bc

a++≥++

333

, for all positive real numbers a, b and c.

* * * , 2005

Problem Problem Problem Problem 8888

Prove that

++

++

+⋅≥

++

b

ac

a

cb

c

ba

a

c

c

b

b

a

2

32

for all positive real numbers a, b and c.

Cezar Lupu, 2006

Problem Problem Problem Problem 9999

Suppose a, b, c are positive real numbers with the sum equal to 1. Prove that:

)(3 222222

cbaa

c

c

b

b

a++≥++ .

Mircea Lascu, 2006

Problem Problem Problem Problem 10101010

Let x, y, z be positive real numbers such that

21

1

1

1

1

1=

++

++

+ zyx.

Prove that 8xyz ≤ 1.

Mircea Lascu, 2006

Problem Problem Problem Problem 11111111

Let a and b be integer numbers. Show that there exists a unique pair of integers x, y so that

(x + 2y – a)2 + (2x – y – b)

2 ≤ 1.

Adrian Zahariuc, 2007

Problem Problem Problem Problem 12121212

Suppose a, b, c are positive real numbers satisfying:

11

1

1

1

1

1≥

+++

+++

++ accbba.

Show that

a + b + c ≥ ab + bc + ca.

Andrei Ciupan, 2007

Page 11: Carte Olimpiada_balcanica-2010

11111111

Problem Problem Problem Problem 11113333

Prove that

|))()((|4

3

3

333

xzzyyxxyzzyx

−−−+≥++

,

for any real numbers x, y, z ≥ 0.

Viorel Vâjâitu, 2007

Problem Problem Problem Problem 14141414

Let a, b, c be positive real numbers with ab + bc + ca = 3. Prove that

abcbacacbcba

1

)(1

1

)(1

1

)(1

1222

≤++

+++

+++

.

Vlad Matei, 2008

Problem Problem Problem Problem 15151515

Determine the maximum value of the real number k such that

(a + b + c) kkcacbba

++

++

+

111,

for all real numbers a, b, c ≥ 0 with a + b + c = ab + bc + ca.

Andrei Ciupan, 2008

Problem Problem Problem Problem 16161616

Let a, b, c > 0 be real numbers with the sum equal to 3. Show that:

abc

c

cab

b

bca

a

+

++

+

++

+

+

3

3

3

3

3

3≥ 3.

Dinu Şerbănescu, 2009

Problem Problem Problem Problem 17171717

Let a, b, c be positive real numbers such that a + b + c ≥ cba

111++ . Prove that

cabcaba

c

c

b

b

a 111++≥++ .

Cezar Lupu, 2009

Problem Problem Problem Problem 18181818

Let A be a non-empty subset of R with the property that for every real numbers x, y, if x + y ∈ A,

then xy ∈ A. Prove that A = R.

Eugen Păltănea, 2001

Page 12: Carte Olimpiada_balcanica-2010

12121212

Problem Problem Problem Problem 19191919

For any positive integer n, let

1212

144)(

2

−++

−+=

nn

nnnf .

Compute the sum f (1) + f (2) + … + f (40).

Titu Andreescu, 2002

Problem Problem Problem Problem 20202020

Five real numbers of absolute values not greater than 1 and having the sum equal to 1 are written

on the circumference of a circle.

Prove that one can choose three consecutively disposed numbers a, b, c, such that all the sums

a + b, b + c and a + b + c are nonnegative. Dinu Şerbănescu, 2003

Problem Problem Problem Problem 21212121

A set of 2003 positive integers is given. Show that one can find two elements such that their sum is

not a divisor of the sum of the other elements.

Valentin Vornicu, 2003

Problem Problem Problem Problem 22222222

Consider the numbers

13 2 −+= nnan

and

)(2 22 nnnnbn −++= ,

for all n = 1, 2, …, 49.

Prove that there are integers A, B so that

2... 49492211 BAbababa +=−++−+− .

Titu Andreescu, 2004

Problem Problem Problem Problem 23232323

Let a < b ≤ c < d be positive integers such that ad = bc and 1≤− ad . Prove that a is a square.

Dinu Şerbănescu, 2004

Problem Problem Problem Problem 24242424

The real numbers a1, a2, …, an satisfy the relation

101)...(... 210021

2100

22

21 =+++++++ aaaaaa .

Prove that |ak| ≤ 10, for all k = 1, 2, …, 100.

Dinu Şerbănescu, 2004

Page 13: Carte Olimpiada_balcanica-2010

13131313

Problem Problem Problem Problem 25252525

Consider the triangular array

0 1 1 2 3 5…

0 1 1 2 3…

2 3 5 8…

4 7 11…

defined by the conditions:

i) on the first two rows, each element starting with the third is the sum of the two preceding

elements;

ii) on the other rows each element is the sum of the two elements placed above on the same

column.

a) Prove that all the rows are defined according to condition i).

b) Consider 4 consecutive rows and let a, b, c, d be the first element in each of these rows,

respectively. Find d in terms of a, b and c.

Dinu Şerbănescu, 2004

Problem Problem Problem Problem 26262626

Let A be a set of positive integers with the properties:

i) if a ∈ A, then all positive divisors of a are elements of A;

ii) if a, b ∈ A and 1 < a < b, then 1 + abc ∈ A.

Prove that if the set A has at least 3 elements, then A = N*.

Valentin Vornicu, 2004

Problem Problem Problem Problem 27272727

Let n > 3 be a positive integer. Consider n sets, each having two elements, such that the intersection

of any two of them is a set with one element. Prove that the intersects of all sets is nonempty.

Sever Moldoveanu, 2005

Problem Problem Problem Problem 28282828

Consider two distinct positive integers a and b having integer arithmetic, geometric and harmonic

means. Find the minimum value of |a – b|. Mircea Fianu, 2005

Problem Problem Problem Problem 29292929

Find all real numbers a and b satisfying

2(a2 + 1)(b

2 + 1) = (a + 1)(b + 1)(ab + 1).

Valentin Vornicu, 2006

Problem Problem Problem Problem 30303030

Show that the set of real numbers can be partitioned into subsets having two elements.

* * *, 2006

Page 14: Carte Olimpiada_balcanica-2010

14141414

Problem Problem Problem Problem 33331111

The set of positive integers is partitioned in subsets with infinite elements each. The question (in

each of the following cases) is if there exists a subset in the partition such that any positive integer has

a multiple in this subset.

a) Prove that if the number of subsets in the partition is finite the answer is yes.

b) Prove that if the number of subsets in the partition is infinite, then the answer can be no (for a

certain partition).

* * *, 2006

Problem Problem Problem Problem 33332222

Find all non-empty subsets A of the set {2, 3, 4, 5, …} so that for any n ∈ A, both n2 + 4 and

][ n + 1 also belong to A.

Lucian łurea, 2007

Problem Problem Problem Problem 33333333

An irrational number x, 0 < x < 1 is called suitable if its first 4 decimals in the decimal

representation are equal. Find the smallest positive integer n such that any real number t, 0 < t < 1 may

be written as a sum of n distinct suitable numbers.

Lucian łurea, 2007

Problem Problem Problem Problem 33334444

Let n ∈ N* and let a1, a2, …, an be positive real numbers so that

222

21

21

1...

11...

n

naaa

aaa +++=+++ .

Prove that for any m = 1, 2 …, n, there exist m numbers among the given ones with the sum not

less than m.

Andrei Ciupan and Flavian Georgescu, 2008

Problem Problem Problem Problem 33335555

Let A be a finite set of positive real numbers satisfying the property:

For any real numbers a > 0, the sets

{x ∈ A | X > a} and

<∈a

xAx1

|

have the cardinals of the same parity.

Show that the product of all elements in A is equal to 1.

Dinu Şerbănescu, 2009

Page 15: Carte Olimpiada_balcanica-2010

15151515

Problem Problem Problem Problem 36363636

Let a and b be positive integers. Consider the set of all non-negative integers n for which the

number nn

ba

++

+

2

1

2

1

is an integer. Show that the set is finite. * * *, 2009

Page 16: Carte Olimpiada_balcanica-2010

16161616

Chapter IIChapter IIChapter IIChapter II

GEOMETRYGEOMETRYGEOMETRYGEOMETRY

Problem Problem Problem Problem 37373737

Let ABC be an arbitrary triangle. A circle passes through B and C and cuts again the lines AB and

AC in D and E, respectively. The projections of the points B and E on CD are denoted by B′ and E′, respectively. The projections of the points D and C on BE are denoted by D′ and C′, respectively.

Prove that the points B′, D′, E′ and C′ lie on the same circle.

Dan Brânzei, 2001

Problem Problem Problem Problem 38383838

Let ABCD be a rectangle. We consider the points E ∈ CA, F ∈ AB, G ∈ BC such that DE ⊥ CA,

EF ⊥ AB and EG ⊥ BC. Solve in the set of rational numbers the equation ACx = EF

x + EG

x.

Dan Brânzei, 2001

Problem Problem Problem Problem 39393939

Let ABCD be a quadrilateral inscribed in the circle O. For a point E ∈ O, its projections K, L, M,

N on the lines DA, AB, BC, CD, respectively, are considered. Prove that if N is the orthocenter of the

triangle KLM for some point E, different from A, B, C, D, then this holds for every point E of the

circle O.

Dan Brânzei, 2001

Problem Problem Problem Problem 44440000

Let ABCDEF be a hexagon with AB || DE, BC || EF, CD || FA and in which the diagonals AD, BE

and CF are congruent. Prove that the hexagon can be inscribed in a circle.

Dan Brânzei, 2001

Problem Problem Problem Problem 44441111

Find the minimal area of a rectangular box of volume strictly greater than 1000, given that the side

lengths are integers.

Dinu Şerbănescu, 2001

PrPrPrProblem oblem oblem oblem 44442222

Let ABCD be a parallelogram of center O. Let M and N be the midpoints of BO and CD,

respectively. Prove that if the triangles ABC and AMN are similar, then ABCD is a square.

Dinu Şerbănescu, 2002

Page 17: Carte Olimpiada_balcanica-2010

17171717

Problem Problem Problem Problem 44443333

Let ABC be an isosceles triangle such that AB = AC and 'A = 20°. Let M be the foot of the altitude

from C and let N be a point on the side AC such that CN = BC2

1. Find the measure of the angle 'AMN.

Dinu Şerbănescu, 2002

Problem Problem Problem Problem 44444444

Let ABCD be a unit square. For any interior points M and N such that the line MN does not contain

any vertices of the square, denote by s(M, N) the least area of a triangle having vertices in the set {A,

B, C, D, M, N}. Find the least number k such that s(M, N) ≤ k, for all such points M, N.

Dinu Şerbănescu, 2002

Problem Problem Problem Problem 44445555

The diagonals AC and BD of a convex quadrilateral meet at O. Let m be the measure of the acute

angle formed by these diagonals. For any angle xOy of measure m, the area inside the angle that is in

the interior of the quadrilateral is constant. Prove that ABCD is a square.

Mircea Fianu, 2002

Problem Problem Problem Problem 46464646

Let C1(O1) and C2(O2) be two circles such that C1 passes through O2. Point M lies on C1 such that

M ∉ O1O2. The tangents from M at C2 meet again C1 at A and B. Prove that the tangents from A and B

at C2 – others than MA and MB – meet at a point located on C1.

Dinu Şerbănescu, 2002

Problem Problem Problem Problem 47474747

Five points are given in the plane such that each of 10 triangles defined by them has area greater

than 2. Prove that there exists a triangle of area greater than 3.

LaurenŃiu Panaitopol, 2002

Problem Problem Problem Problem 48484848

Consider n > 2 concentric circles and two lines D1, D2 which meet at P, a point inside all the

circles. The rays determined by P on the line D1 meet the circles in points A1, A2, …, An and

nAAA ′′′ ...,,, 21 respectively and the rays on D2 meet the circles at points B1, B2, …, Bn and nBBB ′′′ ...,,, 21

(points with the same indices lie on the same circle). Prove that if the arcs A1B1 and A2B2 are equal,

then the arcs AiBi and iiBA ′′ are equal, for all i = 1, 2, …, n.

Dinu Şerbănescu, 2002

Problem Problem Problem Problem 49494949

Let ABC be a triangle and a = BC, b = CA and c = AB be the lengths of its sides. Points D and E lie

in the same halfplane determined by BC as A. Suppose that DB = c, CE = b and that the area of DECB

is maximal. Let F be the midpoint of DE and let FB = x.

Prove that FC = x and

4x3 = (a

2 + b

2 + c

2)x + abc.

Dan Brânzei, 2002

Page 18: Carte Olimpiada_balcanica-2010

18181818

Problem Problem Problem Problem 55550000

Consider a rhombus ABCD with center O. A point P is given inside the rhombus, but not situated

on the diagonals. Let M, N, Q, R be the projection of P on the sides (AB), (BC), (CD), (DA),

respectively. The perpendicular bisectors of the segments MN and RQ meet at S and the perpendicular

bisectors of the segments NQ and MR meet at T. Prove that P, S, T and O are the vertices of a

rectangle.

Mircea Fianu, 2003

Problem Problem Problem Problem 55551111

Two circles C1(O1) and C2(O2) with distinct radii meet at points A and B. The tangent from A to C1

intersects the tangent from B to C2 at point M. Show that both circles are seen from M under the same

angle.

Dinu Şerbănescu, 2003

Problem Problem Problem Problem 55552222

Let E be the midpoint of the side CD of a square ABCD. Consider the point M inside the square

such that 'MAB = 'MBC = 'BME = x. Find the angle x.

LaurenŃiu Panaitopol, 2003

Problem Problem Problem Problem 55553333

Suppose ABCD and AEFG are rectangles such that the points B, E, D, G are collinear (in this

order). Let the lines BC and GF intersect at point T and let the lines DC and EF intersect at point H.

Prove that points A, H and T are collinear.

Mircea Fianu, 2003

Problem Problem Problem Problem 55554444

Two unit squares with parallel sides overlap by a rectangle of area 1/8. Find the extreme values of

the distance between the centers of the squares.

Radu Gologan, 2003

Problem Problem Problem Problem 55555555

Consider a circle of center O and V a point outside the circle. The tangents from V touch the circle

at points T1, T2. Let T be a point on the small arc T1T2 of the circle. The tangent at T intersects the line

VT1 in the point A and the lines TT1 and VT2 intersect in the point B. Let M be the intersection point of

the lines OM and AB. Prove that lines OM and AB are perpendicular.

Mircea Fianu, 2004

Problem Problem Problem Problem 55556666

Let ABC be an acute triangle and let D be a point on the side BC. Points E and F are the projections

of D on the sides AB and AC, respectively. Lines BF and CE meet at point P. Prove that AD is the

bisector line of the angle BAC if and only if lines AP and BC are perpendicular.

Sever Moldoveanu, 2004

Page 19: Carte Olimpiada_balcanica-2010

19191919

Problem Problem Problem Problem 57575757

Consider a triangle ABC with the side lenghts a, b, c so that a is the largest. Prove that the triangle

is right-angled if and only if

2)())(( cbacacababa ++=−++−++ .

Virgil Nicula, 2004

Problem Problem Problem Problem 58585858

Consider the triangle ABC with AB = AC and a variable point M on the line BC so that B is

between M and C. Prove that the sum between the inradius of AMB and the exradius of AMC

corresponding to the angle M is constant.

Virgil Nicula, 2004

Problem Problem Problem Problem 59595959

Let ABC be a triangle inscribed in the circle K and consider a point M on the arc BC that does not

contain A. The tangents from M to the incircle of ABC intersect the circle K at the points N and P.

Prove that if 'BAC = 'NMP, then triangles ABC and MNP are congruent.

Valentin Vornicu, 2004

Problem Problem Problem Problem 66660000

Let M, N, P be the midpoints of the sides BC, CA, AB of the triangle ABC, respectively, and let G

be the centroid of the triangle. Prove that if the quadrilateral BMGP is cyclic and 2BN = AB3 , then

the triangle ABC is equilateral.

BMO shortlist 2004

Problem Problem Problem Problem 66661111

Circles C1 and C2 intersect at points A and B. The tangent line from A to C2 meets C1 at point C and

the tangent line from A to C1 meets C2 at point D. A ray from A, interior to the angle 'CAD, intersects

C1 at M, C2 at N and the circumcircle of the triangle ACD at P. Prove that AM = NP.

Mircea Fianu, 2005

Problem Problem Problem Problem 66662222

Points M and N are given on the sides AD and BC of a rhombus ABCD. Line MC meets the

segment BD at T and line MN meets the segment BD at U. Line CU intersects the side AB at Q and –

finally! – line QT intersects the side CD at P. Show that the triangles QCP and MCN have the same

area.

Mircea Fianu, 2005

Problem Problem Problem Problem 66663333

A point M is given inside an equilateral triangle ABC. Denote by A′, B′, C ′ the projections of the

point M on the sides BC, CA, AB, respectively. Prove that lines AA′, BB′, CC ′ are concurrent if and

only if point M lies on an altitude of the triangle.

Laurentiu Panaitopol, 2005

Page 20: Carte Olimpiada_balcanica-2010

20202020

Problem Problem Problem Problem 64646464

Let ABC be a triangle with BC > CA > AB and let G be the centroid of the triangle. Prove that

'GCA + 'GBC < 'BAC < 'GAC + 'GBA.

Dinu Şerbănescu, 2005

Problem Problem Problem Problem 65656565

Three circles C1(O1), C2(O2), C3(O3) share a common point Q and meet again pairwisely at points

A, B, C. Show that if points A, B, C are collinear then points Q, O1, O2, O3 are cocyclic.

Simpson, 2005

Problem Problem Problem Problem 66666666

Let AB and BC be two consecutive sides of a regular polygon with 9 vertices inscribed in a circle of

center O. Let M be the midpoint of AB and let N be the midpoint of the radius perpendicular to BC.

Find the measure of the angle 'OMN.

* * *, 2005

Problem Problem Problem Problem 66667777

A piece of cardboard has the shape of a pentagon ABCDE in which BCDE is a square and ABE is

an isosceles triangle right-angled at A. Prove that the pentagon can be divided in 2 different ways in

three parts that can be rearranged in order to recompose a right isosceles triangle.

* * *, 2005

Problem Problem Problem Problem 68686868

Let ABC be a triangle right at C and consider points D, E on the sides BC, CA, respectively such

that kCD

AE

AC

BD== . Lines BE and AD meet at point O. Show that 'BOD = 60° if and only if

k = 3 .

Marcel ChiriŃă, 2006

Problem Problem Problem Problem 69696969

In a plane 5 points are given such that all triangles having vertices at these points are of area not

greater than 1. Show that there exists a trapezoid which contains all points in the interior (or on the

sides) and having the area not exceeding 3.

Marcel ChiriŃă, 2006

Problem Problem Problem Problem 77770000

Consider a circle C of center O and let A, B be points on the circle with 'AOB = 90°. Circles

C1(O1) and C2(O2) are internally tangent to C at points A, B, respectively, and – moreover – are tangent

to themselves. Circle C3(O3), located inside the angle 'AOB, is externally tangent to C1, C2 and

internally tangent to C. Prove that points O, O1, O2, O3 are vertices of a rectangle.

* * *, 2006

Page 21: Carte Olimpiada_balcanica-2010

21212121

Problem Problem Problem Problem 77771111

Suppose ABCD is a cyclic quadrilateral of area 8. Prove that if there exists a point O in the plane of

the triangle such that OA + OB + OC + OD = 8, then ABCD is either an isosceles trapezoid or a square.

Flavian Georgescu, 2006

Problem Problem Problem Problem 77772222

Let ABC be a triangle and let A1, B1, C1 be the midpoints of the sides BC, CA, AB, respectively.

Show that if M is a point in the plane of the triangle such that

2111

===MC

MC

MB

MB

MA

MA,

then M is the centroid of the triangle.

Dinu Şerbănescu, 2006

Problem Problem Problem Problem 77773333

Let ABC be a triangle and D a point inside the triangle, located on the median from A. Show that if

'BDC = 180° – 'BAC, then AB ⋅ CD = AC ⋅ BD. Eduard Băzăvan, 2006

Problem Problem Problem Problem 74747474

Consider a trapezoid ABCD with the bases AB and CD so that with the diameters AD and BC are

secant; denote by M and N their common points. Prove that the midsection point of the diagonals AC

and BD belongs to the line MN.

Sever Moldoveanu, 2007

Problem Problem Problem Problem 75757575

Let ABCD be a convex quadrilateral. The incircle ω1 of triangle ABD touches the sides AB, AD at

points M, N respectively, while the incircle ω2 of triangle CBD touches the sides CD, CB at points P,

Q, respectively. Given that ω1 and ω2 are tangent, show that:

a) the quadrilateral ABCD is circumscriptible;

b) the quadrilateral MNPQ is cyclic;

c) the incircles of triangles ABC and ADC are tangent.

Vasile Pop, 2007

Problem Problem Problem Problem 76767676

Let ABC be an acute-angled triangle with AB = AC. For any point P inside the triangle ABC

consider the circle centered at A with radius AP and let M and N be the intersection points of the sides

AB and AC with the circle. Determine the position of the point P such that MN + BP + CP is

minimum.

Francisc Bozgan, 2007

Problem Problem Problem Problem 77777777

Let ABC be a triangle. Points M, N, P are given on the sides AB, BC, CA respectively, so that

CPMN is a parallelogram. Lines AN and MP meet at point R, lines BP and MN meet at point S, while

Q is the intersection point of the lines AN and BP. Show that S[MRQS] = S[NQP].

Mircea Lascu, 2007

Page 22: Carte Olimpiada_balcanica-2010

22222222

Problem Problem Problem Problem 77778888

Circles ω1 and ω2 meet at points A and B. A third circle ω3, which intersects ω1 at points D and B,

is internally tangent to ω2 at point C and tangent to the line AB at point F, and lines DE and AB meet

at point G. Let H be the mirror image of F across G. Find the measure of the angle 'HCF.

Lucian łurea, 2007

Problem Problem Problem Problem 79797979

Let ρ be a semicircle of diameter AB. A parallel line to AB intersects semicircle in C and D so that

points B and C lie on opposite sides of the line AD. The parallel line from C to AD meets ρ again at

point E. Lines BE and CD meet at point F and the parallel line from F to AD intersects AB at point P.

Prove that the line PC is tangent to the semicircle ρ. Cosmin PohoaŃă, 2007

Problem Problem Problem Problem 88880000

Let ABC be a triangle right-angled at A and let D be a point on the side AC. Point E is the mirror

image of A across BD and point F is the intersection of the line CE with the perpendicular line from D

to CB. Show that the lines AF, DE and CB are concurrent.

Dinu Şerbănescu, 2007

Problem Problem Problem Problem 88881111

Let ABC be an acute-angled triangle. Consider the equilateral triangle A′UV, with A′ ∈ (BC), U ∈

∈ (AC), V ∈ (AB) such that UV || BC. Similarly, define points B′ ∈ (AC) and C ′ ∈ (AB). Show that the

lines AA′, BB′ and CC ′ are concurrent.

Vasile Pop, 2008

Problem Problem Problem Problem 88882222

Let ABC be a triangle and D the midpoint of BC. On the sides AB and AC there are points M, N,

respectively, other than the midpoints of these segments, so that AM2 + AN

2 = BM

2 + CN

2 and

'MDN = 'BAC. Prove that A = 90°.

Francisc Bozgan, 2008

Problem Problem Problem Problem 88883333

Consider an acute-angled triangle ABC, the height AD and the point E where the diameter from A

of the circumcircle intersects the line BC. Let M, N be the mirror images of D across the lines AC and

AB. Show that 'EMC = 'BNE.

Dinu Şerbănescu, 2008

Problem Problem Problem Problem 84848484

Let d be a line and let M, N be two points on d. Circles α, β, γ, δ centered at A, B, C, D are tangent

to d in such a manner that circles α, β are externally tangent at M, while circles γ, δ are externally

tangent at N. Moreover, points A and C lie on the same side of line d. Prove that if there exists a circle

tangent to all circles α, β, γ, δ, containing all of them in the interior, then lines AC, BD and d are

concurrent or parallel.

Flavian Georgescu, 2008

Page 23: Carte Olimpiada_balcanica-2010

23232323

Problem Problem Problem Problem 85858585

Let ABCD be a quadrilateral with no two opposite sides parallel. The parallel from A to BD meets

the line CD at point F and the parallel from D at AC meets the line AB at point E. Consider the

midpoints M, N, P, Q of the segments AC, BD, AF, DE respectively. Show that lines MN, PQ and AD

are concurrent.

Dinu Şerbănescu, 2008

Problem Problem Problem Problem 86868686

Consider a rhombus ABCD. Point M and N are given on the line segments AC and BC respectively,

such that DM = MN. Lines AC and DN meet at point P and lines AB and DM meet at point R. Prove

that RP = PD.

Cristinel Mortici, 2009

Problem Problem Problem Problem 87878787

Let ABCD be a quadrilateral. The diagonals AC and BD are perpendicular at point O. The

perpendiculars from O on the sides of the quadrilateral meet AB, BC, CD, DA at M, N, P, Q,

respectively, and meet again CD, DA, AB, BC at M ′, N ′, P ′, Q ′, respectively. Prove that points M, N,

P, Q, M ′, N ′, P ′, Q ′ are concyclic.

Cosmin PohoaŃă, 2O09

Problem Problem Problem Problem 88888888

Consider a regular polygon A0A1…An–1, n ≥ 3, and m ∈ {1, 2, …, n – 1}, m ≠ n/2. For any number

i ∈ {1, 2, …, n – 1}, r(i) be the remainder of i + m at the division by n. Prove that no three segments

AiAr(i) are concurrent.

* * *, 2009

Problem Problem Problem Problem 88889999

Consider K a polygon in plane, such that the distance between any two vertices is not greater than 1.

Let X and Y be two points inside K. Show that there exist a point Z, lying on the border of K, such that

XZ + YZ ≤ 1.

* * *, 2009

Problem Problem Problem Problem 99990000

Show that in any triangle ABC with A = 90° the following inequality holds:

(AB – AC)2(BC

2 + 4AB ⋅ AC)

2 ≤ 2BC6

.

Lucian Petrescu, 2009

Problem Problem Problem Problem 99991111

Let ABC be a triangle and A′ the foot of the internal bisector of angle BAC. Consider the

perpendicular line from A′ on BC.

Define analogously the lines dB and dC. Prove that lines dA, dB and dC are concurrent if and only if

triangle ABC is isosceles.

* * *, 2009

Page 24: Carte Olimpiada_balcanica-2010

24242424

Chapter IIIChapter IIIChapter IIIChapter III

NUMBER THEORYNUMBER THEORYNUMBER THEORYNUMBER THEORY

Problem Problem Problem Problem 99992222

Find n ∈ Z such that the number 5

24

+−

n

n is rational.

Dan Popescu, 2001

Problem Problem Problem Problem 99993333

Three students write on the blackboard next to each other three two-digit squares. In the end, they

observe that the 6-digit number thus obtained is also a square. Find this number!

Mircea Becheanu, 2001

Problem Problem Problem Problem 94949494

Determine all positive integers a < b < c < d with the property that each of them divides the sum of

the other three.

* * *, 2001

Problem Problem Problem Problem 95959595

Let n be a non-negative integer. Find the non-negative integers a, b, c, d, such that

a2 + b

2 + c

2 + d

2 = 7 ⋅ 4n.

LaurenŃiu Panaitopol, 2001

Problem Problem Problem Problem 96969696

Let n ≥ 2 be a positive integer. Find the positive integers x such that

nxxx <+++ ... ,

for any number of radicals.

Ion Dobrotă, 2001

Problem Problem Problem Problem 97979797

Let k, n, p be positive integers such that p is a prime number, k < 1000 and pnk = .

a) Prove that if the equation pxnxk )(100 +=+ has a non-zero integer solution, then p is a

divisor of 10.

b) Find the number of all non-negative solutions of the above equation.

Mircea Fianu, 2002

Page 25: Carte Olimpiada_balcanica-2010

25252525

Problem Problem Problem Problem 98989898

Find all positive integers a, b, c, d such that

a + b + c + d – 3 = ab + cd.

Dinu Şerbănescu, 2002

Problem Problem Problem Problem 99999999

Let n be an even positive integer and let a, b be two coprime positive integers.

Find a and b such that a + b is a divisor of an + b

n.

Dinu Şerbănescu, 2002

Problem Problem Problem Problem 101010100000

Let a be an integer. Prove that for any real number x, x3 < 3, both numbers 23 x− and

3 3xa −

cannot be rational.

LaurenŃiu Panaitopol, 2002

Problem Problem Problem Problem 101010101111

The last four digits of a perfect square are equal. Prove that all of them are zeros.

LaurenŃiu Panaitopol, 2002

Problem Problem Problem Problem 101010102222

Let m, n > 1 be integer numbers. Solve in positive integers

xn + y

n =2

m.

LaurenŃiu Panaitopol, 2002

Problem Problem Problem Problem 101010103333

Let p, q be two distinct primes. Prove that there are positive integers a, b such that the arithmetic

mean of all positive divisors of the number n = paqb is an integer.

LaurenŃiu Panaitopol, 2002

Problem Problem Problem Problem 101010104444

Consider the prime numbers n1 < n2 < … < n31. Prove that if 30 divides 431

42

41 ... nnn +++ , then

among these numbers one can find three consecutive primes.

Vasile Berghea, 2003

ProblProblProblProblem em em em 111105050505

Let n be a positive integer. Prove that there are no positive integers x and y such as

241 +<+<++ nyxnn .

Dinu Şerbănescu, 2003

Problem Problem Problem Problem 111106060606

Let a be a positive integer such that the number an has an odd number of digits in the decimal

representation, for all n > 0. Prove that the number a is an even power of 10.

Vasile Zidaru, 2003

Page 26: Carte Olimpiada_balcanica-2010

26262626

Problem Problem Problem Problem 111107070707

Find all positive integers n for which there exist distinct integers a1, a2, …, an such that

2

......

21 21

21

n

n

aaa

a

n

aa

+++=+++ .

Dinu Şerbănescu, 2004

Problem Problem Problem Problem 111108080808

Let p, q, r be primes and let n be a positive integer such that

pn + q

n = r

2.

Prove that n = 1.

LaurenŃiu Panaitopol, 2004

Problem Problem Problem Problem 111109090909

A finite set of positive integers is called isolated if the sum of the elements in any proper subset is a

number relatively prime with the sum of the elements of the isolated set. Find all nonprime integers n

for which there exist positive integers a, b such that the set A = {(a + b)2, (a + 2b)

2, …, (a + nb)

2} is

isolated.

Gabriel Dospinescu, 2004

Problem Problem Problem Problem 111111110000

Find the greatest integer n, n > 10 such that the remainder of n when divided by each square

between 2 and 2

n is an odd integer.

Adrian Stoica, 2005

Problem Problem Problem Problem 111111111111

Let k, r ∈ N* and let x ∈ (0, 1) be a rational number given in decimal representation:

x = 0,a1a2a3a4…

Show that if the decimals ak, ak+r, ak+2r, ak+3r, … are canceled, the new number thus obtained is still

rational.

Dan Schwarz, 2005

Problem Problem Problem Problem 111111112222

Find all positive integers n and p if p is prime and

n8 – p

5 = n

2 + p

2.

Adrian Stoica, 2005

ProbProbProbProblem lem lem lem 111111113333

For any positive integer n let s(n) be the sum of its digits in decimal representation. Find all

numbers n for which s(n) is the largest proper divisor of n.

LaurenŃiu Panaitopol, 2006

Page 27: Carte Olimpiada_balcanica-2010

27272727

Problem Problem Problem Problem 111114141414

Consider the integers a1, a2, a3, a4, b1, b2, b3, b4 with ak ≠ bk for all k =1, 2, 3, 4. If

{a1, b1} + {a2, b2} = {a3, b3} + {a4, b4},

show that the number |(a1 – b1)(a2 – b2)(a3 – b3)(a4 – b4)| is a square.

Note. For any sets A and B, we denote A + B = {x + y | x ∈ A, y ∈ B}. Adrian Zahariuc, 2006

Problem Problem Problem Problem 111111115555

For a positive integer n denote r(n) the number having the digits of n in reverse order – for

example, r(2006) = 6002. Prove that for any positive integers a and b the numbers 4a2 + r(b) and

4b2 + r(a) can not be simultaneously squares.

Marius Ghergu, 2006

Problem Problem Problem Problem 111116161616

Find all integers n, n ≥ 4 such that ][ n + 1 divides n – 1 and ][ n – 1 divides n + 1.

Marian Andronache, 2007

PPPProblem roblem roblem roblem 111117171717

Solve in positive integers the equation

(x2 + 2)(y

2 + 3)(z

2 + 4) = 60xyz.

Flavian Georgescu, 2007

Problem Problem Problem Problem 111118181818

Determine all positive integers n which can be represented in the form

n = [a, b] + [b, c] + [c, a],

where a, b, c are positive integers.

Note: [p, q] is the lowest common multiple of the integers p and q.

Adrian Zahariuc, 2007

Problem Problem Problem Problem 111119191919

Let p be a prime number, p ≠ 3, and let a, b be integer numbers such that p | a + b and p2 | a

3 + b

3.

Show that p2 | a + b or p

3 | a

3 + b

3.

* * *, 2008

Problem Problem Problem Problem 121212120000

Prove that for any positive integer n there exists a multiple of n with the sum of its digits equal to n.

Mihai Bălună, 2008

Problem Problem Problem Problem 121212121111

Let n ∈ N, n ≥ 2. Consider the integers a1, a2, …, an with 0 < ak ≤ k, for all k = 1, 2, …, n. Given

that a1 + a2 + … + an is an even number, prove that one can choose the signs ‘+’ and ‘–’ such that

a1 ± a2 ± … ± an = 0.

* * *, 2008

Page 28: Carte Olimpiada_balcanica-2010

28282828

Problem Problem Problem Problem 121212122222

A sequence of integers a1, a2, …, an is given so that ak is the number of all multiples of k in this

sequence, for any k = 1, 2, …, n. Find all possible values for n.

Cristian Mangra, 2008

Problem Problem Problem Problem 121212123333

Let a, b be real numbers with the property that the integer part of an + b is an even number, for all

n ∈ N. Show that a is an even integer.

Dinu Şerbănescu, 2002

Problem Problem Problem Problem 111124242424

Find all primes p, q satisfying the equation 2pq – q

p = 7.

Francisc Bozgan, 2008

Problem Problem Problem Problem 111125252525

Find all pairs of integers (m, n), n, m > 1 so that mn – 1 divides n3 – 1.

Francisc Bozgan, 2008

Problem Problem Problem Problem 111126262626

For all positive integers n define an = ���times

3...332n

, where digit 3 occurs n times. Show that the number

a2009 has infinitely many multiples in the set {an | n ∈ N*}.

Cristian Lazăr, 2009

Problem Problem Problem Problem 111127272727

Find all non-negative integers a, b, c, d such that:

7a = 4

b + 5

c + 6

d.

Petre Stângescu, 2009

Problem Problem Problem Problem 111128282828

A positive integer is called saturated if any prime factor occurs at a power greater than or equal to

2 in its factorisation. For example, numbers 8 = 23 and 9 = 3

2 are saturated; moreover, they are

consecutive. Prove that there exist infinitely many saturated consecutive numbers.

* * *, 2009

Page 29: Carte Olimpiada_balcanica-2010

29292929

Chapter IVChapter IVChapter IVChapter IV

COMBINATORICSCOMBINATORICSCOMBINATORICSCOMBINATORICS

Problem Problem Problem Problem 111129292929

Consider a 1 × n rectangle and some tiles of size 1 × 1 of four different colours. The rectangle is

tiled in such a way that no two neighboring square tiles have the same colour.

a) Find the number of distinct symmetrical tilings.

b) Find the number of tilings such that any consecutive square tiles have distinct colours.

Dan Brânzei, 2002

Problem Problem Problem Problem 131313130000

A square of side 1 is decomposed into 9 equal squares of sides 3

1 and the central square is colored

black. The remaining eight squares are analogously divided into nine squares each, and central squares

are colored in black.

Prove that after 1000 steps the total area of the black region exceeds 0.999.

Cristinel Mortici and Costel Chiteş, 2002

Problem Problem Problem Problem 131313131111

An equilateral triangle of side 10 is divided into 100 unit equilateral triangles by lines parallel to

the sides of the triangle. Find the number of (not necessarily unit) equilateral triangles in the

configuration described above such that the sides of the triangles are parallel to the sides of the initial

one.

Dinu Şerbănescu, 2002

Problem Problem Problem Problem 131313132222

Show that one can color all the points of a plane using only two colors such that no line segment

has all points of the same color.

Valentin Vornicu, 2003

Problem Problem Problem Problem 131313133333

Consider a cube and let M, N be two of its vertices. Assign the number 1 to these vertices and 0 to

the other six vertices. We are allowed to select a vertex and to increase with a unit the numbers

assigned to the 3 adjacent vertices – call this a movement.

Prove that there is a sequence of movements after which all the numbers assigned to the vertices of

the cube became equal if and only if MN is not a diagonal of a face of the cube.

Marius Ghergu and Dinu Şerbănescu, 2004

Page 30: Carte Olimpiada_balcanica-2010

30303030

Problem Problem Problem Problem 111133334444

An array 8 × 8 consists of 64 unit squares. Inside each square are written the numbers 1 or –1 so

that in any 2 × 2 subarray the sum of the four numbers equals 2 or –2. Prove that there exist two rows

of the array which are equal.

Marius Ghergu, 2004

Problem Problem Problem Problem 111135353535

In a chess tournament each of the players have played with all the others two games, one time with

the white pieces and then with the black pieces. In each game the winners sets one point and both

players receive 0.5 points if the game ends with draw. At the end of the tournament, all the players end

with the same number of points.

a) Prove that there are two players with the same number of draws.

b) Prove that there are two players with the same number of losses when playing the white.

Marius Ghergu, 2004

Problem Problem Problem Problem 111136363636

A regular polygon with 1000 sides has the vertices colored in red, yellow or blue. A move consists

in choosing to adjacent vertices colored differently and coloring them in the third color. Prove that

there is a sequence of moves after which all the vertices of the polygon will have the same color.

Marius Ghergu, 2004

Problem Problem Problem Problem 111137373737

A country has six cities with airports and two rival flight companies. Any two cities are connected by

flights so that on each route between two cities one may travel with exactly one of the two flight com-

panies. Prove that you can visit 4 cities in a cycle flying with the same air company (that is, there exist

four cities A, B, C, D and a company which operates on the routes A ↔ B, B ↔ C, C ↔ D and D ↔ A).

Dan Schwarz, 2005

Problem Problem Problem Problem 111138383838

A phone company starts a new type of customer service. A new client can choose k phone numbers

in this network which are call-free – regardless if is called or if calling. A group of n students decide to

take advantage of this promotion.

• Show that if n ≥ 2k + 2 then there will exist 2 students which will be charged when speaking.

• Show that if n = 2k + 1 then there exists a way of arranging the free calls so that in this group

everybody speaks free to anyone else.

Valentin Vornicu, 2005

Problem Problem Problem Problem 111139393939

The positive integers from 1 to n2 are placed arbitrarily on squares of an n × n chessboard. Two

squares are called adjacent if they have a common side. Show that two opposite corner squares can be

joined by a path of 2n – 1 adjacent squares so that the sum of the numbers places on them is at least

12

23

+−+

nn

n.

Radu Gologan, 2005

Page 31: Carte Olimpiada_balcanica-2010

31313131

Problem Problem Problem Problem 141414140000

An 7 × 7 array is divided in 49 unit squares. Find all integers n ∈ N* for which n checkers can be

placed on the unit squares so that each row and each line have an even number of checkers.

(0 is an even number, so there may exist empty rows or columns. A square may be occupied by at

most 1 checker.)

Dinu Şerbănescu, 2006

Problem Problem Problem Problem 141414141111

A rectangular cardboard is divided successively into smaller pieces by a straight cut; at each step,

only one single piece is divided in two. Find the smallest number of cuts required in order to obtain –

among others – 251 polygons with 11 sides.

Marian Andronache, 2007

Problem Problem Problem Problem 141414142222

Consider a n × n array divided into unit squares which are randomly colored in black or white.

Three of the four comer squares are colored in white and the fourth is colored in black. Prove that

there exists a 2 × 2 square which contains an odd number of white squares.

Lidia Ilie, 2007

Problem Problem Problem Problem 141414143333

Consider the numbers from 1 to 16. A solitaire game is played in the following manner: the

numbers are paired and each pair is replaced by the greatest prime divisor of the sum of the numbers

in that pair – for example, (1, 2); (3, 4); (5, 6); …; (15, 16) produces the sequence 3, 7, 11, 5, 19, 23, 3,

31. The game continues similarly until one single number is left. Find the greatest possible value of

the number which ends the game.

Adrian Stoica, 2007

Problem Problem Problem Problem 111144444444

Eight persons attend a party, and each participant has at most three others to whom he/she cannot

speak. Show that the persons can be grouped in 4 pairs so that each pair can converse.

Mihai Bălună, 2007

Problem Problem Problem Problem 111144445555

A set of points is called free if there is no equilateral triangle whose vertices are among the points

in the set. Show that any set of n points in the plane contains a free subset of at least n points.

Călin Popescu, 2007

Problem Problem Problem Problem 111146464646

A 8 × 8 square board is divided into 64 unit squares. A “skew-diagonal” of the board is a set of 8

unit squares with the property that each row or column of the board contains only one unit square of

the set. Checkers are placed in some of the unit squares so that each “skew-diagonal” has exactly 2

squares occupied by checkers. Prove that there exist two rows or two columns which contain all the

checkers.

Dinu Şerbănescu, 2007

Page 32: Carte Olimpiada_balcanica-2010

32323232

Problem Problem Problem Problem 111147474747

To obtain a square P of side length 2 cm divided into 4 unit squares it is sufficient to draw 3

squares: P and another 2 unit squares with a common vertex, as shown below:

Find the minimum number of squares sufficient to obtain a square of side length n cm divided into

n2 unit squares (n ≥ 3 is an integer).

* * *, 2009

Problem Problem Problem Problem 141414148888

The plane is divided into a net of equilateral triangles of side length 1, with disjoint interiors. A

checker is placed initially inside a triangle. The checker can be moved into another triangle sharing a

common vertex (with the triangle hosting the checker) and having the opposite sides (with respect to

this vertex) parallel. A path consists in a finite sequence of moves. Prove that there is no path between

two triangles sharing a common side.

Vasile Pop, 2009

Problem Problem Problem Problem 111149494949

Show that there exist (at least) a rearrangement a0, a1, a2, …, a63 of the numbers 0, 1, 2, …, 63,

such that ai – aj ≠ aj – ak, for any i < j < k ∈ {0, 1, 2, …, 63}. * * *, 2009

Problem Problem Problem Problem 111150505050

Let A = {1, 2, …, 2006}. Find the maximal number of subsets of A that can be selected such that

the intersection of any 2 distinct subsets has 2004 elements.

* * *, 2006

Problem Problem Problem Problem 151151151151

Let 1 ≤ m < n be positive integers, and consider the set

M = {(x, y) | x, y ∈ N*, 1 ≤ x, y ≤ n}.

Determine the least value v(m, n) with the property that for any subset P ⊆ M with |P| = v(m, n)

there exist m + 1 elements Ai = (xi, yi) ∈ P, i = 1, 2, …, m + 1, for which the values xi are all distinct,

and yi are also all distinct.

Vasile Pop, 2007

Problem Problem Problem Problem 152152152152

Ten numbers are chosen at random from the set 1, 2, 3, …, 37. Show that one can select four

distinct numbers from the chosen ones so that the sum of two of them is equal to the sum of the other

two.

Vasile Pop, 2008

Page 33: Carte Olimpiada_balcanica-2010

33333333

Problem Problem Problem Problem 153153153153

Let m, n ∈ N* and A = {1, 2, …, n}, B = {1, 2, …, m}. A subset S of the set product A × B has the

property that for any pairs (a, b)(x, y) ∈ S, then (a – x)(b – y) ≤ 0. Show that S has at most m + n – 1

elements.

Dinu Şerbănescu, 2007

Problem Problem Problem Problem 151515154444

In the interior of a circle centered in O a number of 1200 points A1, A2, …, A1200 are considered

such that for every i, j with 1 ≤ i ≤ j ≤ 1200, the points O, Ai and Aj are not collinear. Prove that there

exist the points M and N on the circle, with m('MON) = 30°, such that in the interior of the angle

'MON lie exactly 100 points.

* * *, 2001

Problem Problem Problem Problem 155155155155

Consider a convex polygon with n ≥ 5 sides. Prove that there are at most 3

)52( −nn triangles of

area 1 with the vertices among the vertices of the polygon.

Andrei NeguŃ, 2004

Page 34: Carte Olimpiada_balcanica-2010

34343434

Page 35: Carte Olimpiada_balcanica-2010

35353535

FORMAL FORMAL FORMAL FORMAL SOLUTIONSSOLUTIONSSOLUTIONSSOLUTIONS

Page 36: Carte Olimpiada_balcanica-2010

36363636

Page 37: Carte Olimpiada_balcanica-2010

37373737

Chapter I

ALGEBRA

Solution to Solution to Solution to Solution to Problem Problem Problem Problem 1 1 1 1

Observe that 3 xx < for x ∈ [0, 1]. Thus, we have 3 abcabc < and

3 )1)(1)(1()1)(1)(1( cbacba −−−<−−− .

By the AM-GM inequality, we get

3

3 cbaabcabc

++≤<

and

3

)1()1()1()1)(1)(1()1)(1)(1( 3

cbacbacba

−+−+−≤−−−<−−− .

Summing up, we obtain

13

111)1)(1)(1( =

−++−++−+<−−−+

ccbbaacbaabc ,

as desired.

Solution to Solution to Solution to Solution to Problem Problem Problem Problem 2222

Setting x = a

1, y =

b

1, z =

c

1, we have xyz = 1. The inequality rewrites as

1 + zyxzxyzxy ++

≥++

63.

Since (x + y + z)2 ≥ 3(xy + yz + zx), it follows that

1 + 2)(

91

3

zyxzxyzxy +++≥

++.

It suffices to observe that

1 + zyxzyx ++

≥++

6

)(

92

,

which reduces to

03

1

2

++−

zyx.

Page 38: Carte Olimpiada_balcanica-2010

38383838

Solution to Solution to Solution to Solution to Problem Problem Problem Problem 3333

Using the Chebyshev inequality we derive

(a + b + c)(a2 + b

2 + c

2) ≤ 3(a

3 + b

3 + c

3),

hence a + b + c ≤ 1. On the other hand,

4(ab + bc + ca) – 1 ≥ a2 + b

2 + c

2 ≥ ab + bc + ca,

therefore ab + bc + ca ≥ 1. As

3(ab + bc + ca) ≤ (a + b + c)2 ≤ 1,

we obtain a + b + c ≥ 1, thus a + b + c = 1. Consequently, a + b + c = 1 and

3(ab + bc + ca) = (a + b + c)2,

which imply

a = b = c = 3

1.

Solution to Solution to Solution to Solution to Problem Problem Problem Problem 4444

The given condition rewrites

abc(a + b + c) ≥ ab + bc + ca. From the Cauchy-Schwarz inequality we have

ab + bc + ca ≥ )(3 cbaabc ++ ,

so the conclusion follows.

Solution to Solution to Solution to Solution to Problem Problem Problem Problem 5555

Let s = a + b + c. The given condition rewrites as 1 = (s – a)(s – b)(s – c) = s(ab + bc + ca) – abc,

so ab + bc + ca = cba

abc

++

+1. From the AM-GM inequality we obtain

2

3))()((

2

3))()()((

2

1=+++⋅≥+++++=++ accbbaaccbbacba .

On the other side, 1 = (a + b)(b + c)(c + a) ≥ 8 ⋅ cabcab ⋅⋅ , hence abc ≤ 8

1. Consequently

ab + bc + ca ≤ 4

3

3

2

8

11 =⋅

+ ,

as needed.

Solution to Solution to Solution to Solution to Problem Problem Problem Problem 6666

Wlog, assume that a ≤ b ≤ c.

If a + b ≤ c, then c > 2

3, while a <

2

3 and b <

2

3. In this case (3 – 2a)(3 – 2b)(3 – 2c) < 0 ≤ a

2b

2c

2

and we are done.

If a + b > c, then a, b, c are the side lengths of a triangle with semiperimeter p = 2

3. The inequality

rewrites

8(p – a)(p – b)(p – c) ≤ a2b

2c

2.

Page 39: Carte Olimpiada_balcanica-2010

39393939

Using the formulas S2 = p(p – a)(p – b)(p – c) and abc = 4RS we obtain 8S

2 ≤ 16pR2

S2 or 1 ≤ 3R

2.

This inequality can be proved in many ways, for example from 2p ≤ 33 R, since p =2

3.

Solution to Solution to Solution to Solution to Problem Problem Problem Problem 7777

The inequality rewrites as a4 + b

4 + c

4 ≥ abc(a + b + c). Using successively the inequality

x2 + y

2 + z

2 ≥ xy + yz + zx

we get

a4 + b

4 + c

4 ≥ a

2b

2 + b

2c

2 + c

2a

2 = (ab)

2 + (bc)

2 + (ca)

2 ≥ abc(a + b + c)

as desired.

Solution to Solution to Solution to Solution to Problem Problem Problem Problem 8888

Let yc

bx

b

a== , and

a

c = z. The inequality rewrites successively:

x2 + y

2 + z

2 + 2xy + 2yz + 2zx ≥

+++++zyx

zyx111

2

3 ⇔

⇔ x2 + y

2 + z

2 + 2

++zyx

111 ≥

+++++zyx

zyx111

2

3 ⇔

⇔ 2(x2 + y

2 + z

2) +

zyx

111++ ≥ 3(x + y + z).

From AM-GM inequality we get

2x2 + x

xxx

xxx

x3

13

113 2222 =⋅⋅⋅≥++= .

Summing with the analogously relations we get the conclusion.

Solution to Solution to Solution to Solution to Problem Problem Problem Problem 9999

First solution. Using Cauchy-Schwarz inequality we get:

accbba

cba

ac

c

cb

b

ba

a

a

c

c

b

b

a222

2222

2

4

2

4

2

4222 )(

++

++≥++=++ .

The inequality is reduced to a2 + b

2 + c

2 ≥ 3(a

2b + b

2c + c

2a) or

(a + b + c)(a2 + b

2 + c

2) ≥ 3(a

2b + b

2c + c

2a).

The latter rewrites as 0)(2 ≥−∑ baa , which is obvious.

Second solution. Since a + b + c = (a + b + c)2 = 1, the inequality gives successively:

2222222

)()(3)( cbacbacbaa

c

c

b

b

a++−++≥++−++ ,

or

∑∑ −≥

+− 2

2

)(2 babab

a,

Page 40: Carte Olimpiada_balcanica-2010

40404040

hence

∑∑ −≥− 2

2

)()(

bab

ba.

Since a, b, c ≤ 1, we are done.

Solution to Solution to Solution to Solution to Problem Problem Problem Problem 10101010

Canceling the denominators we get 1 = xy + yz + zx + 2xyz. From AM-GM inequality we get

1 ≥ 4 ⋅ 4 3332 zyx ,

so 1 ≥ (8xyz)3. The conclusion follows.

Solution to Solution to Solution to Solution to Problem Problem Problem Problem 11111111

Solving for a and b the system of equations

=−−

=−+

,2

2

tbyx

sayx

one has

−+−=

+++=

.5

)2()2(

5

)2()2(

tsbay

tsbax

Restating the claim, one has to prove that there exists a unique pair of integers s, t ∈ Z with

s2 + t

2 ≤ 1 so that both numbers (a + 2b) + (s + 2t), (2a – b) + (2s – t) are divisible by 5.

Notice that (a + 2b) + (s + 2t) + 2[(2a – b) + (2s – t)] = 5(a + s), so

x ∈ Z ⇔ y ∈ Z.

Since s2 + t

2 ≤ 1 ⇔ (s, t) ∈ {(0, 0), (1, 0), (0, 1), (–1, 0), (0, –1)}, it follows that

(a + 2b) + (s + 2t) ∈ {a + 2b – 2, a + 2b – 1, a + 2b, a + 2b + 1, a + 2b + 2},

so there is exactly one pair (s, t) with 5 | (a + 2b) + (s + 2t)|, as needed.

Solution to Solution to Solution to Solution to Problem Problem Problem Problem 12121212

The Cauchy-Schwarz inequality gives

(a + b + 1)(a + b + c2) ≥ (a + b + c)2,

so

11

1

)( 2

2

≥++

≥++

++ ∑∑cyccyc bacba

cba.

Then

∑ ∑∑∑ +=++≥+cyc cyccyccyc

abacbaaa 2)(2222

,

and the claim follows immediately.

Page 41: Carte Olimpiada_balcanica-2010

41414141

Solution to Solution to Solution to Solution to Problem Problem Problem Problem 13131313

Let p = |(x – y)(y – z)(z – x)|. Recall the identities:

x3 + y

3 + z

3 – 3xyz = (x + y + z)(x

2 + y

2 + z

2 – xy – yz – zx)

and

x2 + y

2 + z

2 – xy – yz – zx =

2

1[(a – y)

2 + (y – z)

2 + (z – x)

2].

Using AM-GM inequality, we have

(1) x2 + y

2 + z

2 – xy – yz – zx ≥ 3 2

2

3p .

On the other hand, since |x – y| ≤ x + y, |y – z| ≤ y + z and |z – x| ≤ z + x, it follows that

2(x + y + z) ≥ |x – y| + |y – z| + |z – x|. Applying again the AM-GM inequality gives

(2) 2(x + y + z) ≥ 3

2

3p ,

and the claim follows from the inequalities (1) and (2).

Solution to Solution to Solution to Solution to Problem Problem Problem Problem 14141414

Using the AM-GM inequality we derive 3 2)(3

abccabcab

≥++

. As ab + bc + ca = 3, then

abc ≤ 1. Now

abcabc

cabcab

aabcabcaacabacba

1

33

1

)1(3

1

)3(1

1

)(1

1

)(1

12

=++

=≤−+

=−+

=++

=++ ∑∑∑∑∑ ,

as claimed.

Solution to Solution to Solution to Solution to Problem Problem Problem Problem 15151515

Observe that the numbers a = b = 2, c = 0 fulfill the condition ab + bc + ca = a + b + c. Plugging

into the given inequality, we derive that kk ≥

−++

2

1

2

1

4

14 , hence k ≤ 1.

We claim that the inequality holds for k = 1, proving that the maximum value of k is 1. For this,

rewrite the inequality as

11111

)( ≥

++

++

+++

cacbbacabcab ⇔ 1+++≥

+

++∑ cabcabba

cabcab ⇔

⇔ 1+++≥

+

+∑ cabcabcba

ab ⇔ 1≥

+∑ba

ab.

Notice that cba

ab

ba

ab

++≥

+, since a, b, c ≥ 0. Summing over a cyclic permutation of a, b, c we

get

≥+∑ba

ab1=

++

++=

++∑cba

cabcab

cba

ab,

as needed.

Page 42: Carte Olimpiada_balcanica-2010

42424242

Alternative solution. The inequality is equivalent to the following:

kcacbbacba

cbaS ≥

+

++

+++++

++=

111

1.

Using the given condition, we get

=+++

+++++=

++

++

+ ))()((

)(3111 222

accbba

cabcabcba

cacbba

abccabcabcba

cbacabcabcba

−++++

++++++++=

))((

)(2222

= abccba

cbacba

−++

+++++2)(

)1)((,

hence

S = abccba

cba

−++

++2

2

)(

)(.

Now it is clear that S ≥ 1, and the equality occurs for abc = 0. Therefore k = 1 is the maximum

value.

Solution to Solution to Solution to Solution to Problem Problem Problem Problem 16161616

Observe that 3a + bc = a2 + ab + bc + ca = (b + a)(c + a), hence

∑∑ =+++++

=+

+

cyccyc

))(3())()((

1

3

3cba

accbbabca

a

3))()((

)27()9(

))()((

1 222

cyc

2 ≥+++

−−−=−

+++= ∑

accbba

cbaa

accbba ⇔

⇔ ∑∏∑ −+++++−+=−+≤ ))(3)(327(3)3(32722

abccabcabcbaaaa ⇔

⇔ ∑∑ −+≤ abcaba 39272 ⇔ 18 + 3abc ≥ 7(ab + bc + ca).

Multiplying by 3 in the last inequality gives:

2(a + b + c)3 + 9abc ≥ 7(ab + bc + ca)(a + b + c) ⇔

⇔ 2(a3 + b

3 + c

3 + 6abc + )3(79)

22baabcabcba

symsym

∑∑ +≥+ ⇔

⇔ 0))((2223 ≥−+⇔≤ ∑∑∑ bababaa

sym

.

Solution to Solution to Solution to Solution to Problem Problem Problem Problem 17171717

The inequality rewrites as a2c + b

2a + c

2b ≥ a + b + c.

Using Cauchy-Schwarz inequality, we have

2222

)(111

111cba

b

c

a

b

c

a

bac++≥

++

++ .

Page 43: Carte Olimpiada_balcanica-2010

43434343

This implies a2c + b

2a + c

2b ≥ )(1)(

111

)(

111

)( 2

cbacba

cba

cba

cba

cba++⋅≥++

++

++≥

++

++, as claimed.

Solution to Solution to Solution to Solution to Problem Problem Problem Problem 18181818

Let a ∈ A. Then a + 0 ∈ A, hence 0 = 0 ⋅ a ∈ A. For any real number b, b + (–b) = 0 ∈ A, hence

–b2 ∈ A. Thus, A contains all negative numbers. Let c > 0; we have 0<−− cc , so cc −− ∈ A.

It follows that c = ))(( cc −− ∈ A, hence A = R.

Solution to Solution to Solution to Solution to Problem Problem Problem Problem 19191919

Let a = 12 +n and b = 12 −n . Then a2 + b

2 = 4n, ab = 4n

2 – 1 and a

2 – b

2 = 2. Hence

−−+=

−=

+

++= 33

22

3322

)12()12(2

1)( nn

ba

ba

ba

abbanf

and thus

=−++−+−=+++ )7981...3513(2

1)40(...)2()1( 333333fff

364)1729(2

1)19(

2

1)181(

2

1 333 =−=−=−= .

Solution to Solution to Solution to Solution to Problem Problem Problem Problem 20202020

First, we prove that at most two of the sums a + b, b + c, c + d, d + e and e + a can be negative.

Indeed, assume that two non-consecutive sums (say a + b and c + d) are less than 0. Then 1 – e =

= (a + b) + (c + d) < 0 and so 1 < e, a contradiction. Thus, if three sums are negative, then two of them

are not consecutive, which is false. Moreover, if two sums are negative, then these must be

consecutive; in other words, at least three consecutive sums are nonnegative.

Let a + b, b + c, c + d be greater than or equal to zero. If one of the sums d + e or e + a is negative,

then a + b + c = 1 – (d + e) or b + c + d = 1 – (e + a) are at least 1, hence is positive and we are done.

Finally, consider the case when all sums a + b, b + c, c + d, d + e, e + a are positive. Suppose that

a + b + c < 0; then b > (a + b) + (b + c) > 0. Thus, if a + b + c, b + c + d, c + d + e are negative,

then b, c, d are positive and we are done.

a b

c

d

e

Page 44: Carte Olimpiada_balcanica-2010

44444444

Solution to Solution to Solution to Solution to Problem Problem Problem Problem 21212121

Let a1 < a2 < … < a2003 be the elements of the set. We prove the claim by contradiction.

Numbers a1 + a2003, a2 + a2003, …, a2002 + a2003 divide the sum S = a1 + a2 + … + a2003, since

a + b | S – a – b if and only if a + b | S. Hence S = ki(ai + a2003) for all i = 2002,1 , where ki are

integers.

Since ai + a2003 < S < 2003a2003 < 2003(ai + a2003), it follows that ki ∈ {2, 3, …, 2002} for all i =

= 2002,1 . By Pigeonhole principle, there is a pair of indices i ≠ j such that ki = kj, a contradiction.

Solution to Solution to Solution to Solution to Problem Problem Problem Problem 22222222

The key idea is to observe that

an – bn = 2)121(

2

1++−− nnn .

As 112 −+−≥ nnn , it follows that the sum is 524 − .

Solution tSolution tSolution tSolution to o o o Problem Problem Problem Problem 23232323

Consider the integers 0 < m ≤ n < p so that b = a + m, c = a + n and d = a + p.

Then a(a + p) = (a + m)(a + n) and a + p ≤ a + 1 + a2 . As p = m + n + a

mn is an integer, then

a ≤ mn and p ≥ m + n + 1. On the other hand, 1 + a2 ≥ n + m + 1 ⇒ mnnm

a ≥+

≥2

, hence

a ≥ mn. Consequently, a = mn and m = n, so a is a square.

Solution to Solution to Solution to Solution to Problem Problem Problem Problem 22224444

Assume by contradiction that |ak| > 10, for some k. Wlog, let k = 1. Then 21a > 100 and

1... 22100

23

22 <++++ saaa ,

where s = a1 + a2 + … + a100.

On the other hand, the Cauchy-Schwarz inequality yields 21a = (s – a2 – a3 – … – a100)

2 ≤ 100( 100)... 22

10023

22 <++++ saaa ,

a contradiction.

Solution to Solution to Solution to Solution to Problem Problem Problem Problem 25252525

a) In the array below

a b b

d e f

g h i

where i is an element of the third row, observe that i = c + f = (a + b) + (d + e) = (a +d) + (b + c) =

= g + h. The same argument holds for all the other rows, by induction.

b) We prove that d = 2b + 2c – a. Indeed, from the array

Page 45: Carte Olimpiada_balcanica-2010

45454545

a x y z

b t u

c v

d

we derive d = u + v = b + t + (u + z) = 2(b + t) +(x + y)= 2b + 2(t + y) + x – y =

= 2b + 2c + x – (x + a) = 2b + 2c – a.

Solution to Solution to Solution to Solution to Problem Problem Problem Problem 26262626

It is obvious that 1 ∈ A, since 1 is a divisor of any integer. Consider a, b two elements of A with

1 < a < b. Since at least one of a, b or 1 + ab is even, then 2 is an element of A.

We induct on n ≥ 6 to prove that n ∈ A. Assume that k ∈ A for all k = 1, 2, …, n – 1. If n is odd,

then n = 2p + 1 with 1 < 2 < p ∈ A, hence n ∈ A. If n is even, then n = 2p. As above, 2p – 1 and 2p + 1

are elements of A and consequently 1 + (2p – 1)(2p + 1) = 4p2 ∈ A. The first property implies n =

= 2p ∈ A, as needed.

To complete the proof, we need to show that 3, 4, 5 ∈ A. For this, consider a > 2 an element of A.

Then 1 + 2 ⋅ a ∈ A, 1 + 2(1 + 2a) = 3 + 4a ∈ A and 1 + (1 + 2a)(3 + 4a) = 4 + 10a + 8a2 ∈ A. If a is

even, then 4 | 4 + 10a + 8a2 and so 4 ∈ A. If a is odd, then choose a to be 4 + 10a + 8a

2 and again

4 ∈ A. Next, as 1 < 2 < 4 ∈ A we have 1 + 2 ⋅ 4 = 9 ∈ A and so 3 ∈ A. Finally, 7 = 1 + 2 ⋅ 3 ∈ A, 15 =

= 1 + 2 ⋅ 7 ∈ A, hence 5 ∈ A and we are done.

Solution to Solution to Solution to Solution to Problem Problem Problem Problem 27272727

Let A = {a, b} and B = {a, c} be two of the given sets. A third set C intersect both of them, so C is

{b, c} or {a, d}. In the first case, a fourth set cannot have in common with each of the first three sets

exactly one element, so C = {a, d}.

Any set from the other n – 3 > 1 sets that do not contain a should contain simultaneously the

elements b, c and d, a contradiction. Hence the claim is proved.

Solution to Solution to Solution to Solution to Problem Problem Problem Problem 22228888

Let 2

bax

+= and

ba

aby

+=

2. Then ab = xy and a

2 – a(a + b) + ab = a

2 – 2ax + xy = 0, so x(x – y) =

= (a – x)2 is a square.

Let d be the greatest common divisor of x and y and write x = dx′, y = dy′ with x′, y′ ∈ Z and (x′, y′) =

= 1. Then d 2x(x′ – y′) is a square and (x′, x′ – y′) = 1, so both x′ and x′ – y′ are squares; write x′ = m2

and x′ – y′ = n2.

Recall that the geometric mean xyab = is an integer, so xy = d 2x′y′ = d

2m

2 (m

2 – n

2) is a

square. Consequently m2 – n

2 = p

2, where p is an integer.

To conclude, to this point we have x = dm2, y = dp

2 and since a = x ± )( yxx − , b =

= x )(( yxx −∓ , it follows that a = dm(m ± n) and b = dm(m ∓ n). Therefore |a – b| = 2dmn with the

minimum of 30 obtained for d = 1, m = 5, n = 3 – don’t forget the Pythagorean triples!

Page 46: Carte Olimpiada_balcanica-2010

46464646

Solution to Solution to Solution to Solution to Problem Problem Problem Problem 29292929

First solution. Consider the given equation as quadratic in a:

a2(b

2 – b + 2) – a(b + l)

2 + 2b

2 – b + 1 = 0.

The discriminant is ∆ = –(b – 1)2(7b

2 – 2b + 7), hence we have solutions only for b = 1. It follows

that a = 1.

Second solution. Using Cauchy-Schwarz inequality, we get 2(a2 + 1) ≥ (a + 1)

2, 2(b

2 + 1) ≥ (b + 1)

2

and (a2 + 1)(b

2 + 1) ≥ (ab + 1)

2. Multiplying, we obtain 2(a

2 + 1)(b

2 + 1) ≥ (a + 1)(b + 1)(ab + 1),

hence the equality case occur in all the inequalities, so a = b = 1.

Solution to Solution to Solution to Solution to Problem Problem Problem Problem 30303030

For example, consider R \ Z partitioned in {–x, x} and Z in {2n, 2n + 1}.

Another example: split R into disjoint intervals [2n, 2n + 1), with n ∈ Z. Then take pairs (x, x + 1)

from each interval [2n, 2n + 1), with x ∈ [2n, 2n + 1).

Solution to Solution to Solution to Solution to Problem Problem Problem Problem 33331111

a) Let Ak be the partition classes, with k = 1, 2, …, r. Assuming that the answer is no, there exist nk,

k = 1, 2, …, r, such that no multiples of nk is in Ak. But n1n2…nr lies in one of the sets Ak and is

multiple of any nk, false.

b) We exhibit a partition for which the answer is no.

Let Ak be the set of all numbers written only with the first k primes at any positive power;

moreover, put 1 ∈ A1. Fixing k, the number p1p2…pk+1 have no multiples in Ak.

Solution to Solution to Solution to Solution to Problem Problem Problem Problem 33332222

We claim that A = {2, 3, 4, 5, …}.

Let m be the smallest element of the set A. Since ][ m + 1 ∈ A, we have m ≤ ][ m + 1 ≤ m + 1,

which gives m = 2.

Notice that ]4[ 2 +n = n for all n ≥ 2. Indeed, n2 ≤ n

2 + 4 < (n + 1)

2 = n

2 + 2n + 1, for all n ≥ 2.

Using both hypothesis, we have

n ∈ A ⇒ n2 + 4 ∈ A ⇒ ]4[ 2 +n + 1 ∈ A ⇒ n + 1 ∈ A.

The conclusion follows by induction.

Solution to Solution to Solution to Solution to Problem Problem Problem Problem 33333333

At first, we look for a lower margin of n. The number 0.1111 can be written as a sum of n distinct

suitable numbers, all starting with 0.0000…, therefore lower than 0.0001. Hence if 0.1111 = a1 + a2 +

+ … + an, with ak < 0.0001, then 0.1111 < n ⋅ 0.0001, or n > 1111. Thus n is at least 1112.

We claim that 1112 is the requested number. Let t ∈ (0, 1) be a real number. If t > 0.1111, choose a

suitable number of the form y = 0.xxxx… so that y < t ≤ y + 0.1111. The other suitable numbers will

have the form 0.0000…, so they are different from y. We have 0 < t – y ≤ 0.1111. Because

0 < 1112

1111.0

1112≤

− yt< 0.0001, the first four decimals of the number u =

1112

yt − are all equal to 0. Choose

an irrational number e, small enough not to change the first four decimals of the numbers u + e,

Page 47: Carte Olimpiada_balcanica-2010

47474747

u + 2e, …, u + 1111e, and such that all – plus y + u + e – are left irrationals. Then

t – y = (u + e) + (u + 2e) + (u + 3e) + … + (u + 1111e) +

⋅⋅−

2

11121111 eu .

As previously stated, number e was selected so that all summands are irrational, suitable numbers

with the first four decimals 0 and paiwisely distinct – in other words, different from the last one.

Consequently,

t = (y + u + e) + (u + 2e) + (u + 3e) + … + (u + 1111e) +

⋅⋅−

2

11121111 eu .

The number y + u + e starts with 0.xxxx, while the others with 0.0000, so they are suitable. Since t

is now represented as required, the proof is concluded.

Solution to Solution to Solution to Solution to Problem Problem Problem Problem 33334444

It is clear that we need to prove that a1 + a2 + … + an ≥ n. Let us notice that this is enough: let m =

= 1, 2, …, n and assume that any selection of m numbers from the given ones has the sum less than 1.

Then add the n inequalities

,...

,...

,...

11

132

21

maaa

maaa

maaa

mn

m

m

<+++

<+++

<+++

+

to get n(a1 + a2 + … + an) < nm, which is a contradiction.

Back to the top, let g be the geometrical mean of the numbers a1, a2, …, an and suppose that

a1 + a2 + … + an < n.

By AM-GM inequality, we have g ≤ 1...21 <

+++

n

aaa n , while

1 > 2

222

2121 1

1...

11

...

gn

aaa

n

aaa nn ≥+++

=+++

,

which gives g > 1, a contradiction.

Solution to Solution to Solution to Solution to Problem Problem Problem Problem 33335555

The main idea is to show that x ∈ A if and only if x

1 ∈ A. Since A is finite, one may pair the

elements of A in

x

x1

, , with number 1 left alone if belongs to A. Then it is obvious that the product

of all elements is equal to 1.

Let b > 1 be an arbitrary real number. Applying the hypotesis for a = b and a = b

1 we find that the

sets {x ∈ A | x > b} and

<∈b

xAx1

| have the cardinals of the same parity, and also the sets

Page 48: Carte Olimpiada_balcanica-2010

48484848

>∈b

xAx1

| and

=<∈ b

b

xAx1

1| have the cardinals of the same parity.

This implies that the sets L =

>∈b

xAx1

| \ {x ∈ A | x > b} =

∈∈ bb

xAx ,1

| and R =

= {x ∈ A | x < b} \

<∈b

xAx1

| =

∈∈ bb

xAx ,1

| have the cardinals of the same parity. But the

set L may have at most one element which is not in R and vice versa, which shows the sets L and R

have the same number of elements. Thus both numbers b and b

1 are or aren’t elements of A, as

claimed.

Solution to Solution to Solution to Solution to Problem Problem Problem Problem 36363636

The number

nn

ba

++

+

2

1

2

1 is an integer if and only if 2

n divide (2a + 1)

n + (2b + 1)

n.

For n = 2k, 2n = 4

k divides (2a + 1)

n + (2b + 1)

n = (4a

2 + 4a + 1)

k + (4b

2 + 4b + 1)

k ≡ 2 (mod 4),

implying k ≤ 1, hence the set of even integers satisfying the claim is finite.

For n = 2k + 1, one has

(2a + 1)n + (2b + 1)

n = (2a + 1 + 2b + 1)[(2a + 1)

2k – (2a + 1)

2k–1(2b + 1) +

+ (2a + 1)2k–2

(2b + 1)2 – … + (2b + 1)

2k].

The second factor is a sum of odd summands, each summand being odd, thus being also an odd

integer. The number 2a + 2b + 2 has only a finite number of divisors of the form 2k, therefore the

claim is proved.

Page 49: Carte Olimpiada_balcanica-2010

49494949

Chapter II

GEOMETRY

Solution to Solution to Solution to Solution to Problem Problem Problem Problem 37373737

First solution. Let I be the intersection point of the lines BE and CD. The quadrilaterals BD′B′D and CE ′C ′E are cyclic, hence 'BDB′ = 'B′D′I and 'CEC ′= 'IE ′C ′. Since BDEC is also cyclic,

'BDB′= 'CEC ′. It follows that 'B′D′I = 'IE′C ′, so B′D′E′C ′ is a cyclic quadrilateral.

Second solution. Using the power of a point theorem, one has:

IB′ ⋅ ID = ID′⋅ IB IC ′⋅ IE = IE′⋅ IC IE ⋅ IB = ID ⋅ IC From these one easily obtains IB′⋅ IE′= ID′⋅ IC ′, which proves that the quadrilateral B′D′E′C ′ is

cyclic, using the reciprocal of the power of a point theorem.

Solution to Solution to Solution to Solution to Problem Problem Problem Problem 38383838

Denote AD = a, AB = b. We have AC2 = a

2 + b

2, CE = b

2/AC, AE = a

2/AC and EF/a = AE/AC =

= a2/AC. It follows that EF =

2

3

AC

a and, analogously, EG =

2

3

AC

b. Thus, the equation is equivalent to

(a2 + b

2) = (a

3x + b

3x)2.

Notice that x = 3

2 is a solution. Moreover, for a = b the solution is unique.

Suppose a ≠ b; wlog a > b. Set k = a

b ∈ (0, 1) to obtain (1 + k

2)3x = (1 + k

3)x.

A

D E

E′ D′

C ′

B′

B C

Page 50: Carte Olimpiada_balcanica-2010

50505050

If x < 3

2, then k

3x > k

2 ⇒ 1 + k

3x > 1 + k

2 > 1 ⇒ (1 + k

3x)2 > (1 + k

2)2 > (1 + k

2)3x.

If x < 3

2, we use a similar argument.

Thus, the only solution is x = 3

2.

Solution to Solution to Solution to Solution to Problem Problem Problem Problem 39393939

Let F and G be the projections of E on the diagonals BD and AC. From Simson’s theorem, it

follows that the triplets of points (K, L, F), (M, N, F), (K, G, N) and (M, L, G) are collinear. The point

N is the orthocenter of the triangle KLM if and only if KL ⊥ MN and ML ⊥ KN. Let F ′ and G ′ be the

points in which EF and EG intersect the second time the circle. We have KF || AF ′ and MG || CF ′. Thus KL ⊥ MN is equivalent to AF ′⊥ CF ′ and then to O ∈ AC. Similarly, ML ⊥ KN is equivalent to

O ∈ BD. Thus, ABCD is a rectangle. It is easy to see that in this case, N is the orthocenter of the

triangle KLM for any position of the point E.

Solution to Solution to Solution to Solution to Problem Problem Problem Problem 44440000

We first notice that ABDE is an isosceles trapezoid. The segments AB and DE have the same

perpendicular bisector. Let O and R the center and radius of the circumcircle of the triangle ABC. One

can see that the perpendicular bisectors of DE and CF also pass through O, hence O is the center of the

circle circumscribed around DCF, with radius R′. Finally, since ACDF is an isosceles trapezoid, it

follows that R = R′.

Solution to Solution to Solution to Solution to Problem Problem Problem Problem 44441111

Let a ≤ b ≤ c the lengths of the parallelepiped’s. We have abc ≥ 1001 and c ≥ 11. By analysing the

cases c ∈ {11, …, 21} one finds that a = 8, b = 9 and c = 14 is the solution.

Solution to Solution to Solution to Solution to Problem Problem Problem Problem 44442222

From the similarity of the triangles AMN and ABC, we obtain

(1) AC

AN

AB

AM=

and

(2) 'MAN = 'BAC or 'BAM = 'CAN.

The relations (1) and (2) imply the similarity of the triangles BAM and CAN. Hence, we obtain the

proportions:

(3) CN

BM

AC

AB

AN

AN==

and 'ABM ≡ 'ACN. The last equality implyies that ABCD is a rectangle.

To conclude the proof, notice that BM = 4

1BD =

4

1AC and CN =

2

1AB. Hence the last equality in

Page 51: Carte Olimpiada_balcanica-2010

51515151

(3) becomes AB

AC

AC

AB

2= , that is 2AB

2 = AC

2 = AB

2 + BC

2, which proves that ABCD is a square.

Solution to Solution to Solution to Solution to Problem Problem Problem Problem 44443333

Let P the midpoint of BC. Since MP is a median in the right-angled triangle MBC, it follows that

PB = MP = PC = CN.

The point R is considered such that PCNR is a parallelogram (in fact a rhombus). Notice that

'RPM = 'RPB – 'MPB = 'ACB – (180° – 2'MBC) = 60°,

and BP = MP, therefore MPR is an equilateral triangle. Hence MR = RP = RN and 'MRN = 'MRP +

+ 'PRN = 60° + 80° = 140°. Then 'RMN = 'RNM = 20°, 'ANM = 20° + 80° = 100° and the required

angle 'AMN is equal to 60°.

Solution to Solution to Solution to Solution to Problem Problem Problem Problem 44444444

Let E, F be the midpoints of AD and BC repectively and let M, N be the midpoints of OE and OF.

It is easy to check that 8

1),( =NMS , therefore

8

1≥k . We claim that

8

1=k .

Notice that area[BMC] + area[AMD] = 2

1, for any interior point M. We may assume that N is an

interior point of the triangle AMD. Therefore area[AND] + area[ANM] + area[DNM] + area[BMC] =

= 2

1. Consequently, one of the summands exceeds

8

1. The conclusion follows.

A

M

B P C

N R

20°

20°

A B

C D

E F M N

A B

C D

M

N

Page 52: Carte Olimpiada_balcanica-2010

52525252

Solution to Solution to Solution to Solution to Problem Problem Problem Problem 44445555

Consider 'AOD = m ≤ 90°. As the angles 'AOD and 'BOC are equal to m, we find area[AOD] =

= area[BOC]. It follows

mCOBOmDOAO sin2

1sin

2

1⋅⋅=⋅⋅ ,

that is DO

BO

CO

AO= . Since 'AOB = 'DOC, the triangles AOB and DOC are similar and consequently

AB is parallel to DC.

Draw line EF that contains O, such that 'AOE = 'COF = m and E ∈ (AB), F ∈ (DC). The triangles

AOE and COF are similar and have the same area, that is they are congruent. It follows that AO = OC

and in the same way BO = OD, implying AD || BC. Moreover area(COF) = area(BOC), and since

ABCD is a parallelogram, we find area[BOC] = area[DOC]. Hence D = F and m = 'COF = 'DOF =

= 'BOC = 90°. We thus proved that ABCD is a rhombus.

To conclude, consider the bisector lines OM and ON of angles 'AOD and 'DOC respectively,

where M ∈ (AD), N ∈ (DC). It is easy to check that 'MON = m = 90°, whence area[MON] =

= area[AOD]. Thus area[DON] = area[ACM], that is area[AOM] = area[DOM] = 2

1area[AOD]. It

follows that OM is a median in the triangle AOD, that is AO = OD, which proves that the rhombus

ABCD is a square.

Solution to Solution to Solution to Solution to Problem Problem Problem Problem 46464646

Since O2 is at equal distances from the tangents MA and MB, it follows that MO2 is a bisector line

of the angle 'AMB or of the exterior angle defined by MA and MB.

In each case one obtains O2A = O2B.

Reflecting the figure with respect to the line O1O2, the circles C1 and C2 remain fixed, A reflects in

B and M reflects in N. It is obvious that NA, the reflected of MB is tangent to C2 and the same is valid

for NB. Observe that N is on C1, proving thus the claim.

A

N B

M

O1 O2

A E B

C F D

O

m

m

m

Page 53: Carte Olimpiada_balcanica-2010

53535353

Solution to Solution to Solution to Solution to PPPProblem roblem roblem roblem 47474747

Denote by A, B, C, D, E the five given points. If the pentagon ABCDE is concave, we can suppose

that D is located inside the triangle ABC or inside the quadrilateral ABCD.

In first case area[ABC] = area[ABD] + area[DBC] + area[DAC] > 6 > 3.

In the second case, D is inside the one of triangles BCE, ACE, ABC or ABD. Suppose, without loss

of generality, that D is inside to the triangle BCE. Then

area[BCE] ≥ area[BDC] + area[CDE] > 4 > 3.

Consider now the case when ABCD is a convex pentagon. Let M and N be the intersection points of

BE with AC and AD respectively.

The following result will be useful.

Lemma. Let PQRS be a quadrilateral and T a point on the side PQ. Then

area[TRSR] ≥ min(area[PRS], area[QSR]).

The proof consists of simply observing that the distance from T to SR is bounded up and below by

the distances from P and Q to SR.

In our case, suppose that BM ≥ 3

1BE, which yields BM ≥

3

1ME. Then

area[BDE] = area[BDM] + area[MDE] ≥ 2

1area[MDE] + area[MDE] =

= 2

3area[MDE] ≥

2

3min(area[CDE], area[ADE]) >

2

3 ⋅ 2 = 3.

The case when NE ≥ 3

1BE is similar. It is left to consider the case when MN ≥

3

1BE. We then

have:

area[AMN] ≥ 3

1area[ABE] >

3

2,

area[MND] ≥ 3

1area[BED] >

3

2,

and

area[MCD] ≥ min[area[BCD], area[ECD] > 2.

Summing up, we conclude

area[ACD] > 2 + 3

2

3

2+ > 3,

and the proof is complete.

A

B

C D

E N

M

Page 54: Carte Olimpiada_balcanica-2010

54545454

Solution to Solution to Solution to Solution to Problem Problem Problem Problem 48484848

Let O be the common center of the n circles and α = A1B1 = A2B2 (the arcs are directly oriented).

Rotate the figure around the center O by angle α such that A1, A2 become B1, B2 respectively. The

above rotation R maps lines into lines, that is R(D1) = D2, since D1 = A1A2 and D2 = B1B2. Moreover,

any circle Ci is invariant under the rotation. As R(Ai) lies on D2 and on Ci, we get that R(Ai) = Bi, that is

AiBi = α.

In the same way we get ii BAR ′=′)( and iiBA ′′ = α. This concludes the proof.

Solution to Solution to Solution to Solution to Problem Problem Problem Problem 44449999

Let DECB be the quadrilateral of maximal area. It is easy to prove that 'DBE = 'DCE = 90°.

It follows FB = FC = 2

DE = x and that the quadrilateral DBCE is cyclic. By Ptolemey’s theorem

we have DC ⋅ BE = BC ⋅ DE + DB ⋅ CE.

Squaring, we get

(4x2 – b

2)(4x

2 – c

2) = (2ax + bc)

2,

that is

4x2 – (b

2 + c

2)x

2 + b

2c

2 = 4a

2x

2 + 4abcx + b

2c

2.

From this we obtain 4x3 = (a

2 + b

2 + c

2)x + abc, as desired.

Solution to Solution to Solution to Solution to Problem Problem Problem Problem 55550000

Firstly, observe that triangles RSN and QSM are congruent (S.S.S.), hence 'PMS = 'PNS and

'PQS = 'PQS. It follows that P, S, M, N are concyclic and P, S, Q, R are concyclic.

On the other hand, as 'BNP + 'BMP = 180°, points B, N, S, M are concyclic, thus P, S, N, B, M are

points on the circle C1(O1) of diameter BP. Likewise, points P, S, Q, D, R he on the circle C2(O2) of

diameter DP.

Since PS is the common chord of the circles C1 and C2, lines PS and O1O2 are perpendicular. As O1

and O2 are the midpoints of the segments BP and DP, lines O1O2 and BD are parallel, so PS ⊥ BD and

B

A R

M

N

C

Q

D S O

T P

A

E

B C

F D

Page 55: Carte Olimpiada_balcanica-2010

55555555

then PS || AC. Likewise, PT || BD and consequently PS ⊥ PT.

Furthermore, because O1O2 is middle line in the triangle PBD one find that S lies on the segment

BD. Analogously, T ∈ (AC). Thus, PSOT is a rectangle.

Solution to Solution to Solution to Solution to Problem Problem Problem Problem 55551111

We have to prove that 2'O1MA = 2'O2BM, which is equivalent to

(1) BM

BO

AM

AO 21 = .

The lenght of the common chord AB is equal to 2O1A ⋅ sin2

1AB = 2O1A ⋅ sin'BAM, regardless if

AB is the small arc or the great arc AB. Similarly, AB = 2O2B ⋅ sin'ABM, hence

(2) BAM

BO

ABM

AO

'' sinsin

21 = .

By the Law of Sines in the triangle ABM we derive that

(3) BAM

MB

ABM

MA

'' sinsin= .

From the relations (2) and (3) we obtain BM

BO

AM

AO 21 = , as desired.

Solution to Solution to Solution to Solution to Problem Problem Problem Problem 55552222

Observe that 'MAB + 'MBA = 'MBC + 'MBA = 90°, hence

'AMB = 90°.

Let F be the midpoint of the side AB. Then MF = FA = FB = 2

1AB, so 'MBF = 'MBF. It follows

that

'EMF = 'EMB + 'BMF = 'MAB + 'MBA = 90°.

A

B

M

O1 O2

Page 56: Carte Olimpiada_balcanica-2010

56565656

In the right triangle MEF, the leg MF is equal to 2

1EF. hence 'MEF = 30°. We obtain 'MBF =

= 2

1'MFA =

2

1MEF = 15° and x = 75°.

Solution to Solution to Solution to Solution to Problem Problem Problem Problem 55553333

Lines CD and FG meet at M and lines BC and EF meet at N. As DHEA and FHCT are cyclic

quadrilaterals, it follows that

'FTC = 180° – 'FHC = 'DAE and 'DAH = 'DEH.

Since 'DMG = 90° – 'FTC = 90° – 'DAE = 'DAG, it follows that the quadrilateral ADGM is

cyclic. Hence 'DAM = 'FGE and consequently 'MAH = 'DAM + 'DAH = 'FGE + 'DEH = 90°.

Likewise, 'NAH = 90° and therefore points M, A, N are collinear.

In the triangle TMN, point H is the orthocenter. Thus A, H, T lie on the altitude of the triangle, as

desired.

Solution to Solution to Solution to Solution to Problem Problem Problem Problem 55554444

Let MNPQ be the rectangle at the intersection of the unit squares with

centers A and B. Set MN = x and PQ = y, hence

8

1=xy , x, y ∈ [0, 1].

The parallel from A to MN intersects the parallel from B to NP at point C.

It is easy to observe that AC = 1 – x and BC = 1 – y, so

T

H F

G

M A

N

B

C

E D

A B

C D

M

A

B

C

Q P

M N

Page 57: Carte Olimpiada_balcanica-2010

57575757

AB2 = (1 – x)

2 + (1 – y)

2 = x

2 + y

2 – 2(x + y) + 2 =

= x2 + 2xy + y

2 – 2(x + y) –

4

1 + 2 + (x + y)

2 – 2(x + y) +

4

7 =

= 4

3)1( 2 +−+ yx .

It follows that the minimal value of the distance between the centers A and B is equal to 2

3 and it

is obtained for x + y = 1, xy = 8

1; i.e. x =

4

22+, y =

4

22− or x =

4

22−, y =

4

22+.

To find the maximal value of AB observe that 0 ≤ (1 – x)(1 – y) = 1 – x – y + xy = 8

9 – (x + y), so

x + y ≤ 8

9. On the other hand, we have x + y ≥

2

12 =xy , therefore

2

1 – 1 ≤ x + y – 1 ≤

8

1. As

22

12

1

8

1

−≤

, we find that AB

2 ≤ 2

2

122

4

9

4

312

2

1

−=−=+− . Thus AB ≤ 2 –

2

1, with

equality when x = y = 22

1.

Consequently, 2

12

2

3−≤≤ AB .

Solution to Solution to Solution to Solution to Problem Problem Problem Problem 55555555

The approach of the problem is to see no circles in the figure. Instead, recall that a quadrilateral

ABCD is orthogonal if and only if

(1) AB2 – CD

2 = AD

2 + BC

2.

Using successively the Pytagoras theorem we have

BA2 – 2

2BT = BA2 – (BO

2 – 2

2OT ) = BA2 – (BO

2 – 2

1OT ) =

= BA2 – (BA

2 – 2

1AT ) = 21AT = AO

2 – 2

1OT = OA2 – 2

2OT .

so the conclusion follows from the relation (1).

Solution to Solution to Solution to Solution to Problem Problem Problem Problem 55556666

Let a, b, c, x, y be the lengths of the sides BC, CA, AB, BD, DC, respectively and let A′ be the foot

of the altitude from A in the triangle ABC. Notice that x + y = a. Due to Ceva theorem the claim is

equivalent to

DC

BD

AC

AB

EB

AE

FA

CF

DC

BD=⇔=⋅⋅ 1 .

As CF = y cos C, FA = b – y cos C, BE = x cos B, AE = c – x cos B, BA′ = c cos B and A′C = b cos C,

the equivalence rewrites

cy(c – x cos B) = bx(b – y cos C) ⇔ xb = cy.

Page 58: Carte Olimpiada_balcanica-2010

58585858

Indeed, we have

ab

cbabxyxb

ac

bcacxyyc

22

2222

2222 −+

−=−+

− ⇔

⇔ a(c2y – b

2x) = xy(c

2 – b

2) ⇔ c

2y(a – x) = b

2x(a – y) ⇔

⇔ c2y

2 = b

2x

2 ⇔ cy = bx,

as claimed.

Solution to Solution to Solution to Solution to Problem Problem Problem Problem 57575757

Squaring both sides of the equality yields

22222 )())((2 cbacaabaa ++=−+−+ .

It is easy to observe that the equality holds if a2 = b

2 + c

2. To prove the converse statement, assume

that a2 > b

2 + c

2. Then 22 ba − > c and 22 ca − > b, hence

))((2 2222 caabaa −+−+ > 2(a + b)(a + c) =

= 2a2 + 2(ab + bc + ca) > a

2 + b

2 + c

2 + 2(ab + bc + ca) = (a + b + c)

2,

false.

The case a2 ≤ b2

+ c2 leads similarly to a contradiction and we are done.

Solution to Solution to Solution to Solution to Problem Problem Problem Problem 58585858

The idea is to prove that the sum of the radii is equal to the altitude h from A of the triangle ABC; a

hint is to assume that line MA is parallel to BC.

The Stewart relation gives AM2 ⋅ BC + AC2

⋅ MB = AB2 ⋅ MC + MB ⋅ MC ⋅ BC, so AM

2 ⋅ BC =

= AB2(MC – MB) + MB ⋅ MC ⋅ BC, hence AM

2 = AB

2 + MB ⋅ MC. Let r be the inradius of triangle

AMB and R the exradius of triangle AMC corresponding to the angle 'M.

Since

ABMBAM

AMBr

++

⋅=

area2,

and

ABMBMA

AMCR

−+

⋅=

area2,

then

ABMBAM

MB

h

r

++= ,

and

ABMBAM

MC

h

R

−+= .

Thus

r + R = h ⇔ MB(MA + MB – AB) + MC(MA + MB + AB) = (MA + MB + AB)(MA + MB – AB) ⇔

⇔ MB(MA + MB – AB) = (MA + MB + AB)(MA – AB) ⇔

⇔ MB ⋅ MC + MB(MA – AB) = MA2 – AB

2 + MB(MA – AB) ⇔ MA

2 = AB

2 + MB ⋅ MC,

so the claim holds.

Page 59: Carte Olimpiada_balcanica-2010

59595959

Solution to Solution to Solution to Solution to Problem Problem Problem Problem 59595959

Let Q be the intersection point of the line segments AB and MP. The tangents from A and M to the

incircle are equal (to r ⋅ cot 2

A). Moreover, the tangents from Q to the incircle are equal, so AQ = MQ.

This implies 'QMA = 'QAM, so the arcs AP and BM are equal. In the trapezoid APBM, the diagonals

AB and MP are equal, and likewise AC = MN. This concludes the proof.

Solution to Solution to Solution to Solution to Problem Problem Problem Problem 66660000

By the Power of a point theorem we have AG ⋅ AM = AP ⋅ AB, so 4MA2 = 3AB

2 and thus AM =

= 2

3AB = BN. Then AG = GB, so the median GP is also an altitude in the triangle AGB. This implies

'BPG = 90°, and since BMGP is cyclic, 'GMA = 90°. It follows that BC = CA and AB = AC, so the

triangle is equilateral.

Solution to Solution to Solution to Solution to Problem Problem Problem Problem 66661111

Consider the case when the ray is interior to the angle 'BAD. Then

'CMP = 'MCA + 'CAM = 'MAD + 'CAM = 'CAB.

On the other side,

'CPM = 'CDA,

since both subtend the chord AC in the circumcircle ACD.

From the similarity of the triangles ACD and MCP we derive that

AD

MP

AC

MC= .

Furthermore, 'ACM = 'NAD and 'CAM = 'ADN, so 'ACM ~ 'DAN.

Hence

AC

MC

AC

AN= ,

thus MP = AN and AM = NP, as claimed.

Solution to Solution to Solution to Solution to Problem Problem Problem Problem 66662222

The distance from M to the line BC is equal to the distance from Q to the line DC, since ABCD is a

rhombus. Hence it suffices to prove that CP = CN or DP = BN.

Using Thales theorem we obtain UM

NU

UC

QU

UD

BU== , hence NQ || MC and likewise MP || CQ.

Triangles MDP and BQC are similar, so BT

DT

BC

MD

BQ

DP== . Analogously,

DC

BQ

MD

BN= . It follows that

MD

BN

MD

DP= , therefore DP = BN.

Page 60: Carte Olimpiada_balcanica-2010

60606060

Solution to Solution to Solution to Solution to Problem Problem Problem Problem 66663333

Denote AB′ = x1, BC ′ = y1, CA′ = z1, A′B = x2, B′C = y2, C ′A = z2. From the Ceva theorem we have

x1y1z1 = x2y2z2.

As AB′2 = AP2 – PB′2 (and the analogous relations) we obtain by summation

22

22

22

21

21

21 zyxzyx ++=++ .

Since x1 + y2 = x2 + z1 = y1 + z2 = AB and x1 + x2 + y1 + y2 + z1 + z2 = 3 ⋅ AB, it follows that

x1 + y1 + z1 = x2 + y2 + z2.

Thus

x1 + y1 + z1 = = x2 + y2 + z2; 222222111111 zyzxyxzyzxyx ++=++ ; x1y1z1 = x2y2z2,

so {x1, y1, z1} = {x2, y2, z2}.

An easy check shows that the claim holds.

Solution to Solution to Solution to Solution to Problem Problem Problem Problem 64646464

It suffices to prove that 'GAC < 'GBA and 'GBC < 'GAC, given that CA > AB and BC > CA.

Since the two claims are equivalent, we will prove the first inequality.

First approach

The reflection B′ of the point B across AG lies on the parallel line through C at AG. Observe that

point C is further than B′ from the perpendicular bisector d of the line segment AG, since d intersects

line BC at a point located at the left of point B (with respect to point C) and we are done.

Second approach

As area[GBA] = area[GAC], the inequality 'GAC < 'GBA (< 90°!) is equivalent to c ⋅ mb > b ⋅ mc.

Squaring both sides yields b2 ⋅ (2a2

+ 2c2 – b

2) > c

2 ⋅ (2a2

+ 2b2 – c

2), hence (b

2 – c

2)(b

2 + c

2 – 2a

2) > 0,

which is implied by a > b > c.

Solution to Solution to Solution to Solution to Problem Problem Problem Problem 65656565

Set Q1, Q2, Q3 the mirror images of Q across O1, O2, O3, in other words consider the diametrically

opposite points of Q in the circles C1(O1), C2(O2), C3(O3).

It is easy to check that the claim is equivalent to the fact that points Q, Q1, Q2, Q3 lie on the same

circle.

An inversion of pole Q maps the line A – B – C to a circle A′ – B′ – C′ passing through Q, while points

Q1, Q2, Q3 map to points 321 ,, QQQ ′′′ , which are the projection of Q to lines A′B′, B′C′, C′A′. By Simpson

theorem, points 321 ,, QQQ ′′′ are collinear, thus points Q, Q1, Q2, Q3 lie on the same circle, as needed.

Solution to Solution to Solution to Solution to Problem Problem Problem Problem 66666666

Let P be the intersection point of the circle with the radius ON. Triangle AOP is isosceles at A and

'AOP = 60°, so 'ANO = 90°. On the other side 'AMO = 90°, so AMNO is cyclic. Hence

'OMN = 'OAN = 30°.

Solution to Solution to Solution to Solution to Problem Problem Problem Problem 66667777

The first decomposition is obtain as follows:

Consider the midpoint M of the side DE and C ′ the intersection point of the lines BE and CM. The

right isosceles triangle obtain by reassembling is CAC ′, since ∆ABC = ∆AEC ′ and ∆CMD = ∆MEC ′.

Page 61: Carte Olimpiada_balcanica-2010

61616161

For the second decomposition consider N the midpoint of DE and let A′ be the intersection point of

the lines BD and AM. The right isosceles triangle obtain by reassembling is CAA′, as ∆ANE = ∆NDA′ and ∆ABC = ∆CDA′.

Solution to Solution to Solution to Solution to Problem Problem Problem Problem 68686868

Consider the rectangle ACDP. The hypothesis rewrites as kAP

AE

DP

BD== , so 'APE = 'BPD and

'APD = 'EPB. Moreover, PB

PD

PE

AP= , hence ∆PAD ~ ∆PEB.

It follows that 'DAP = 'PEB, so APOE is cyclic and hence 'BOD = 'AOE = 'APE.

The claim is proved by the following chain of equivalences:

'BOD = 60° ⇔ tan'BOD = 3 ⇔ tan'APE = 3=AP

AE ⇔ k = 3 .

Solution to Solution to Solution to Solution to Problem Problem Problem Problem 69696969

Denote A, B, C, D, E the given points and suppose ABC is the triangle having the maximal area.

The distance from D to BC is not greater than the distance from A to BC, hence D – and similarly E –

are located between the parallel line from A to BC and its mirror image across BC. Applying the same

reasoning for AB and AC, one obtains a triangular (bounded) region A1B1C1 – with ABC as median

triangle – in which all points must lie.

Points D and E are located in at most 2 of the triangles A1BC, AB1C, ABC1, hence one of the

trapezoids APA1B1, BCB1C1 or ABA1B1 contains all points. Since the area of the trapezoids is 3 times

the area of ABC, hence not greater than 3, the conclusion is proved.

Solution to Solution to Solution to Solution to Problem Problem Problem Problem 77770000

Let R, r1, r2 be the radii of the circles C, C1, C2 and let r = R – r1 – r2. Consider the point P so that

OO1PO2 is a rectangle. From the tangency conditions we get OO1 = R – r1, OO2 = R – r2 and O1O2 =

= r1 + r2 = R – r. It is sufficient to prove that the circle centered at P with radius r is C3.

To prove this, notice that O1P = OO2 = R – r2 = r + r1, O2P = OO1 = R – r1 = r + r2, and OP =

= O1O2 = R – r, so the 3 tangency conditions are fulfilled.

Solution to Solution to Solution to Solution to Problem Problem Problem Problem 77771111

Let α be the measure of the angle determined by the diagonals.

Since 8 = OA + OB + OC + OD ≥ AC + BD ≥ 2 ⋅ ==α⋅⋅⋅≥⋅ SBDACBDAC 22sin2 8, we

get AC = BD = 1 and α = 90°. The claim follows from a simple arch subtraction.

Solution tSolution tSolution tSolution to o o o Problem Problem Problem Problem 77772222

Let A2, B2, C2 be the mirror images of the point M with respect to points A1, B1, C1. The given

condition shows that MA = MA2, MB = MB2, MC = MC2. From the parallelograms AMBC2, BMCA2,

AMCB2 we derive that MA = MA2 = BC2 = B2C, MB = MB2 = AC2 = CA, and MC = MC2 = PA2 = AP2.

It follows that MA2BC2, MA2CB2 and MB2AC2 are also parallelograms, therefore A, M and A2 are

collinear. The conclusion is now obvious.

Page 62: Carte Olimpiada_balcanica-2010

62626262

Solution to Solution to Solution to Solution to Problem Problem Problem Problem 77773333

Let E be the mirror image of D across the midpoint of the side BC. We notice that DBEC is a

parallelogram and ABEC is cyclic. The equality of the areas of triangles ABE and ACE implies

AB ⋅ BE = AC ⋅ CE. We are left only to notice that CE = BD and BE = CD.

Solution to Solution to Solution to Solution to Problem Problem Problem Problem 74747474

Let Q be the intersection point of the diagonals, T the second point of intersection of the line AC

with the circle C1 of diameter AD and S the second point of intersection of the line BD with the circle

C2 of diameter BC. Then 'ATD = 'BSC = 90°, so DT and SC meet in the orthocenter H of the triangle

DQC. Denote by C3 the circle DCTS.

The radical axis of the circles C1, C2 is MN, the radical axis of the circles C1, C3 is DT, while the

pair of circles C2, C3 has SC as radical axis, hence the radical center of the three circles is H.

The line segment MN is the common chord of the circles C1 and C2, thus perpendicular to the line

passing through the centers, which is in fact the middle line of the trapezoid. As H ∈ MN, then

MH || DC, and since QH || DC the conclusion follows.

Solution to Solution to Solution to Solution to Problem Problem Problem Problem 75757575

Let T ∈ BD be the tangency point of incircles of the triangles ABD and CBD. Notice that BM =

= BT = BN and DN = DT = DM.

a) We have AB + CD = AM + MB + CP + PD = AN + BQ + CQ + DN = AD + BC, so the

quadrilateral ABCD is circumscriptible.

b) Triangles AMN, DNP, CQP, BQM are isosceles, so

'QMN + 'NPQ = 360° – ('AMN + 'BMQ + 'QPC + 'NPC) =

= 360° – 2

1(4 ⋅ 180° – A – B – C – D) = 180°,

hence the quadrilateral MNPQ is cyclic.

c) Let U be the point where the side AC touches the incircle of triangle ABC. Since AB – BC =

= AD – DC, then AU = 22

DCACADBCACAB −+=

−+, so U is also the tangency point of the side

AC with the incircle of triangle ADC, as needed.

Solution to Solution to Solution to Solution to Problem Problem Problem Problem 76767676

For a fixed point P inside the given triangle consider the point Q on bisector line of BC so that

AQ = AP. The parallel line d from Q to BC separates the arc MN and the side BC, so d meets the line

segment [BP] at a point, say S. The triangle’s inequality gives SP – PC ≥ SC, so BP + PC ≥ BS + SC.

On the other hand, with an argument frequently refer to as Heron’s problem we have BS + SC ≥ BQ + + QC, so BP + PC is minimum if P = Q.

Let T be the midpoint of the segment MS. Notice that triangle AMQ is isosceles and MT is an

altitude in this triangle, hence MT = QZ, where Z is the foot of the altitude from Q onto AC. Then

MN + BQ + QC = 2(MT + CQ) = 2(CQ + QZ) is minimum when CZ ⊥ AC. Consequently, the

required point is the orthocenter of the triangle ABC, which belongs to the interior of the triangle,

since it is an acute-angled one.

Page 63: Carte Olimpiada_balcanica-2010

63636363

Solution to Solution to Solution to Solution to Problem Problem Problem Problem 77777777

Let AB

AMk = . Using Thales Theorem, MP || BC yields k

AC

AP= ,while MN || AC implies k

MN

MS= .

On the other hand, kPM

PR

BC

CN== .

Setting S = area[MNP], we have

S

NPR

PM

RP

MN

MS

S

MSP ][area][area=== ,

hence area[MSP] = area[NPR]. Subtracting area[RPQ] from both sides of the latter equality we get the

conclusion.

Solution to Solution to Solution to Solution to Problem Problem Problem Problem 77778888

Line AB is the radical axis of the circles ω1 and ω2, and line DE is the radical axis of the circles ω1

and ω3, hence point G is the radical center of the three circles. Since the radical axis of the circles ω3

and ω2 is the tangent line at C to these circles, it follows that the tangents from G to ω – 3 are GF and

GC. Then GF = GC and GH = GF, so the triangle HCF is right-angled at C. Therefore 'HCF = 90°.

Solution to Solution to Solution to Solution to Problem Problem Problem Problem 79797979

A “special” position occurs when 'CAB = 60°, when C = E = F. In this case the claim is obvious.

Consider the case 'CAB > 60°, where E belongs to the small arc CD and F lie on the segment CD.

Notice that 'PFC = 'ADC = 'BCD, hence the trapezoid PBFC is isosceles. On the other hand, as

CD || AB and CE || AD, it follows that the arcs AC, BD, DE are equal. Then 'EFC = CE + BD = CE +

+ DE = CD = 'P'CD, where P' ∈ PC, C ∈ (PP'). The last equality proves the conclusion.

Slight changes in notations are required for the case 'CAB < 60°.

Solution to Solution to Solution to Solution to Problem Problem Problem Problem 88880000

Line BC meet DF, AE at points T, G respectively. Using Ceva’s theorem, it suffices to prove that

1=⋅⋅DC

AD

GA

EG

FE

CF,

since only one or all points D, F, G lies on the sides of the triangle AEC.

Observe that 'BAD = 'BED = 'BTD = 90°, so points A, C, E, T, D lies on the circle of diameter

BD. Then 'FDE = 'TBE not

= α and 'TDC = 'ABC not

= β. Moreover, DE = DA and AB = BE. The law

of sinuses gives

β==

α sinsin,

sinsin

FC

CFD

DC

EFD

DEFE

''.

and since sin'EFD = sin'CFD we have

(1) α

β⋅=sin

sin

DA

DC

FE

CF.

On the other hand,

Page 64: Carte Olimpiada_balcanica-2010

64646464

β==

α sinsin,

sinsin

AG

AGB

BA

EGB

BEEG

''.

hence

(2) β

α=

sin

sin

GA

EG.

Multiplying the inequalities (1) and (2) concludes the proof.

Alternative solution. Let H be the intersection point of the lines DF and AE. The claim is equivalent

to HE

AH

GE

AG= , in other words the pairs of points A, E and G, H are harmonical conjugates.

Since I is the midpoint of the segment AE, the claim reduces to IG ⋅ IH = IA2.

The segment AI is an altitude in the right-angled triangle ABD, so AI2 = ID ⋅ IB.

Angles 'HTB and 'HIB are right, so the points H, T, I, B are cocyclic. It follows that 'DHI = 'IBG

and further, ∆DHI ~ ∆IBG. Hence IB

IG

IH

ID= , so ID ⋅ IB = IH ⋅ IG, the conclusion is now obvious.

Solution to Solution to Solution to Solution to Problem Problem Problem Problem 88881111

Consider the equilateral triangle BCA1, constructed in the exterior of triangle ABC. Then points A,

A′, A1 are collinear through the homothety of center A which map points U, V in B, C, respectively

Since AA1, BB1, CC1 concur in the Fermat-Torricelli point of the triangle ABC, the claim is proved.

Solution to Solution to Solution to Solution to Problem Problem Problem Problem 88882222

Let E and F be the midpoint of the sides AC and AB and let P be the mirror image of D across E.

The relation AM2 + AN

2 = BM

2 + CN

2 gives

2222

2222

++

−=

−+

+ NE

bFM

cNE

bFM

c,

hence c ⋅ FM = b ⋅ NE. Then AB

NE

AC

FM= , so

EP

NE

FD

FM= . Since 'MFD = 'NEP, we get ∆MFD ~

~ ∆NEP, which implies 'MDF = 'NPE. On the other hand 'MDN = 'BAC = 'FDE, so 'MDF =

= 'NDE.

Now the triangle NPD is isosceles and NE is a median in this triangle, so NE ⊥ DP, in other words

A = 90°.

Solution to Solution to Solution to Solution to Problem Problem Problem Problem 83838383

Observe that AD = AN = AM and 'AND = 'AMC = 90°, due to the reflections across AB and AC. It

is known that AD and AE are isogonal cevians, that is 'BAD = 'EAC. Then 'NAE = 'NAB + 'BAE =

= 'BAD + 'BAE = 'EAC + 'DAC = 'EAC + 'CAM = 'EAM and consequently ∆NAE = ∆EAM. It

follows that 'ENA = 'EMA, so 'BNE = 90° – 'ENA = 90° – 'EMA = 'EMC, as desired.

Solution to Solution to Solution to Solution to Problem Problem Problem Problem 84848484

Let a, b, c, d be the radii of the circles α, β, γ, δ. It suffices to prove that d

c

b

a= , in other words the

Page 65: Carte Olimpiada_balcanica-2010

65656565

ratio b

a is constant while point M varies on line d.

Let R and S be the midpoints of the arcs determined by d on the fifth circle K, the one tangent

simultaneously to α, β, γ, δ, and let N be on the same side of d as A. Denote by A1 and B1 the tangency

point of α and β to K, respectively. Observe that points A1, M, R are collinear – via the homothety

which maps circle a onto circle K – and similarly points B1, M, S are collinear. Since RS is a diameter

of K, angles 'RA1S and 'SB1R are right. If lines B1R and A1S meet as point V, then M is the orthocenter

of the triangle VRS. Notice that d ⊥ RS. hence V ∈ d; denote by O the intersection point of d and RS.

Lines A1S and B1R intersect the circles α and β at points U and Z respectively. Since 'RA1S =

= 'SB1R = 90°, the segments UM and ZM are diameters in circles α, β, so SO

RO

ZM

UM

b

a== . The latter

ratio is constant, as claimed.

Solution to Solution to Solution to Solution to Problem Problem Problem Problem 85858585

Let O be the midpoint of AD, R be the intersection point of lines AC and BD and S be the

intersection point of lines AF and DE. Since N and Q are the midpoints of the sides DB and DE of the

triangle DBE, we have O ∈ NQ and similarly O ∈ MP. Moreover, as DRAS is a parallelogram, the

diagonal RS passes through the midpoint O of the other diagonal, AD. Now apply Desargues Theorem

for triangles NRM and SPQ, given that O lies simultaneously on lines NQ, MP, RS and we are done.

Solution to Solution to Solution to Solution to Problem Problem Problem Problem 86868686

The diagonal AC is the internal bisector for the angles 'BAD and 'BCD, so 'BAC = 'CAD =

= 'BCA = 'ACD. Observe that the triangles CMN and CMD have 'MCN = 'MCD and MN = MD,

hence their circumradius are equal and consequently CNMD is cyclic. It follows that 'MDP = 'MCN =

= 'RAP, implying that ARPD is cyclic. Then 'DRP = 'PAD = 'RAP = 'MDP, hence RP = PD, as

needed.

Solution to Solution to Solution to Solution to Problem Problem Problem Problem 87878787

A well-known result states that the quadrilateral MNPQ is cyclic. For this, notice that the

quadrilaterals BMON, CNOP, DPOQ and AQOP have a pair of opposite right angles, hence all of

them are cyclic. It follows that 'OMN = 'OBN, 'OMQ = 'OAQ, 'OPQ = 'ODQ and 'OPN = 'OCN.

Summing up yields 'QMN = 'QPN, so MNPQ is cyclic, as claimed.

Next we’ll show that the points M ′, N ′, P′, Q′ lie on the circumcircle K of MNPQ. As an external

angle, we get 'NQ′O = 'Q′CO + 'COQ′. Since CNOP is cyclic, then 'Q′CO = 'NPO. On the other

hand we have 'Q′OP = 'ODQ, because the sides are perpendicular, and 'ODQ = 'OPQ, as DPOQ is

cyclic. Therefore 'NQ′O = 'NPO + 'OPQ = 'NPQ, implying that Q′ belongs to the circumcircle of

MNPQ. In the same manner we find that N ′, P′, Q′ he on K, which concludes the proof.

Solution to Solution to Solution to Solution to Problem Problem Problem Problem 88888888

Since m ≠ 2

n, no segment AiAr(i) contain the center O of the polygon A0A1…An–1.

Page 66: Carte Olimpiada_balcanica-2010

66666666

The chords AiAr(i) are equal, since all subtend m from the n equal arcs in which the vertices A0, A1,

…, An–1 divide the circumcircle of the polygon.

Consequently, the distances from O to each of the segments AiAr(i) are equal, hence these segments

are all tangent to a non-degenerate circle centered at O.

Since through a point one cannot draw more than two tangents to a circle, it follows that there are

no three concurrent segments of the form AiAr(i).

Solution to Solution to Solution to Solution to Problem Problem Problem Problem 88889999

Since the points X and Y are inside the polygon K, the line XY with the border of K contains two

points Z and Z ′, such that X, Y ∈ (ZZ ′). The diameter of the polygon K is not greater than 1, hence |ZZ ′| ≤ 1. To prove this, let AB the side

of K containing Z and let CD the side containing Z ′, where A, B, C, D are vertices of the polygon, not

necessarily distinct. Notice that AC, AD, BC, BD ≤ 1. In a triangle MNP, a cevian MS, S ∈ (NP)

satisfies the inequality MS < max(MN, MP). Therefore

ZZ ′ < max(ZC, ZD) < max(max(CA, CB), max(DA, DB)) ≤ 1,

as claimed.

To conclude, observe that (XZ + YZ) + (XZ ′ + YZ ′) = (XZ + XZ ′) + (YZ + YZ ′) = 2ZZ ′ ≤ 2, hence

at least one of the sums XZ + YZ, XZ ′+YZ ′ is less than or equal to 1.

Solution to Solution to Solution to Solution to Problem Problem Problem Problem 99990000

Let a = BC

AB and

BC

ACb = .

Then a2 + b

2 = 1 and the inequality rewrites as (a – b)

2(1 + 4ab)

2 ≤ 2 and furthermore

(1 – 2ab)(1 + 4ab)2 ≤ 2.

Put x = 2ab to obtain (1 – x)(1 + 2x)2 ≤ 2 or 4x

3 – 3x + 1 ≥ 0, which is equivalent to

(2x – 1)2(x + 1) > 0.

The latter is obvious, so we are done.

Solution to Solution to Solution to Solution to Problem Problem Problem Problem 99991111

If the triangle ABC is isosceles, the claim is obvious. To prove the converse, observe that

A′B2 + B′C2

+ C ′A2 = A′C2

+ B′A2 + C ′B2

.

Using the standard notation one has

A′B = cb

ac

+, A′C =

cb

ab

+

and the analogous formulas. The above equality rewrites as

∑∑ +=

+ 2

22

2

22

)()( cb

ba

cb

ca,

hence

0)(2

=+

−∑cb

bca.

Clearing the denominators one obtains (a – b)(b – c)(c – a)(a + b + c)2 = 0, implying that ABC has

at least two congruent sides.

Page 67: Carte Olimpiada_balcanica-2010

67676767

Chapter III

NUMBER THEORY

Solution to Solution to Solution to Solution to Problem Problem Problem Problem 99992222

Suppose 2

2

5

24

b

a

n

n=

+

−, where a and b are coprime integers. One obtains

n = 22

2

22

22

4

225

4

52

ab

b

ab

ab

−+−=

+.

Since gcd(b2, 4b

2 – a

2) = 1, it follows that 4b

2 – a

2 divides 22.

Observe that 4b2 – a

2 ≡ 0 or 4b

2 – a

2 ≡ 3 (mod 4), hence we have either 4b

2 – a

2 = –1, or 4b

2 – a

2 =

= 1. The first case leads to b = 0, which is impossible. In the second case, we obtain 2b – a = 1 and

2b + a = 11, hence a = 5, b = 3 and n = 13.

Solution to Solution to Solution to Solution to Problem Problem Problem Problem 99993333

Suppose that the number obtained is n2 = abcdef , where efcdab ,, ∈ {16, 25, 36, 49, 64, 81}.

Since 161616 ≤ n2 ≤ 818181, it follows that 402 ≤ n

2 ≤ 904. Thus, n = 100x + 10y + z and x ≥ 4, z ≥ 1.

By the squaring algorithm we obtain x2 = ab . Also:

(l00x + 10y + z)2 = 10

4ab + 10

3c + 10

2d + 10e + f,

hence

2 ⋅ 103xy + 2 ⋅ 10

2xz + 10

2y

2 + 2 ⋅ 10yz + z

2 = 10

3c + 10

2d + 10e + f.

Two cases arise:

a) x = 4, y ∈ {0, l};

For y = 1, ab = 16, cd = 81, by using the above equality we get a contradiction.

For y = 0, ab = 16 and 8z ⋅ 102 + z

2 = 10

2efcd + . Since the representation of a number is unique

we get cd = 8z and since ef is a two digit number, it follows that 8z ∈ {16, 64}. Therefore, z = 8 and

n = 408.

b) x > 4 and y = 0. We obtain (200x + z)z = 102

efcd + . As before we obtain cd = 2xz, ef = z2,

z ≥ 4. It follows that x = 8, z = 4, hence n = 804.

In conclusion, the students can obtain the numbers 4082 or 804

2.

Solution to Solution to Solution to Solution to PrPrPrProblem oblem oblem oblem 94949494

Since a + b + c < 3d and d | a + b + c, it follows that a + b + c = d or a + b + c = 2d. Suppose that

a + b + c = d. Since a | b + c + d = 2d – a, it follows that a | 2d and, similarly, b | 2d, c | 2d. Let

Page 68: Carte Olimpiada_balcanica-2010

68686868

2d = ax = by = cz, where x > y > z > 2. We obtain 2

1111=++

zyx.

If z ≥ 6, then 2

1

6

1

6

1

6

1111=++<++

zyx, so there are no solutions.

If z = 5, then 5

311=+

yx, and we obtain y = 3, again not possible.

If z = 4, then 4

111=+

yx, and we obtain the solutions (k, 4k, 5k, 10k) and (k, 2k, 3k, 6k), with k ∈ N.

If z = 3, then 6

111=+

yx, and we obtain the solutions (k, 6k, 14k, 21k), (k, 3k, 8k, 12k), (k, 2k, 6k, 9k)

and (2k, 3k, 10k, 15k), with k ∈ N.

Now, suppose that a + b + c = 2d. Analogously, we obtain that a, b, c | 3d, hence 3d = ax = by = cz

with x > y > z > 3 and 3

2111=++

zyx. Then z ≥ 4, y ≥ 5, x ≥ 6, thus

3

2

60

37

4

1

5

1

6

1111<=++≤++

zyx, so

there are no solutions in this case.

Solution to Solution to Solution to Solution to Problem Problem Problem Problem 95959595

For n = 0, we have 22 + 1

2 + 1

2 + 1

2 = 7, hence (a, b, c, d) = {2, 1, 1, 1) and all permutations.

If n ≥ 1, then a2 + b

2 + c

2 + d

2 = 0 (mod 4), hence the numbers have the same parity. We analyse

two cases.

a) The numbers a, b, c, d are odd. We write a = 2a′+ 1 etc. We obtain:

4a′(a′ + 1) + 4b′(b′+ 1) + 4c′(c′ + 1) + 4d ′(d ′+ 1) = 4(7 ⋅ 4n–1 – 1).

The left hand side of the equality is divisible by 8, hence 7 ⋅ 4n–1 must be even. This happens only

for n = 1. We obtain a2 + b

2 + c

2 + d

2 = 28, with the solutions (3, 3, 3, 1) and (1, 1, 1, 5).

b) The numbers a, b, c, d are even. Write a = 2a′ etc. We obtain

a ′2 + b′2 + c′2 + d ′2 = 7 ⋅ 4n–1

so we proceed recursively.

Finally, we obtain the solutions (2n+1

, 2n, 2

n, 2

n), (3 ⋅ 2n, 3 ⋅ 2n, 3 ⋅ 2n, 2n), (2n, 2n, 2n, 5 ⋅ 2n), n ∈ N,

and all permutations.

SSSSolution to olution to olution to olution to Problem Problem Problem Problem 96969696

Clearly x ≤ n2, so let x = n

2 – p, with p > 0. If the number of radicals is 2, we obtain that x ≤ n

2 – n.

It is easy to check using induction that all x ≤ n2 – n verify the inequality regardless the number of

radicals.

Solution to Solution to Solution to Solution to PrPrPrProblem oblem oblem oblem 97979797

a) By squaring the members of the equation we get k + 100x = n2p + 2nxp + x

2p, or 100 = p(2np + x).

The conclusion follows from the fact that p is a prime number.

b) If p = 2, then 50 = 2n + x and 0 ≤ n2 ≤ 25. Since n

2 =

2

k

p

k= < 500, it follows that n ≤ 22 and we

Page 69: Carte Olimpiada_balcanica-2010

69696969

have 23 solutions.

If p = 5, then 20 = 2n + x and 0 ≤ n ≤ 10. Notice that n2 =

5

k

p

k= < 200 for any n ≤ 10, therefore we

have other 11 solutions. We have 34 solutions in all.

Solution to Solution to Solution to Solution to Problem Problem Problem Problem 98989898

We have ab + ad = 2(a + b + c + d) – 6, so (a – 2)(b – 2) + (c – 2)(d – 2) = 2.

Assuming that a is the smallest number among a, b, c, d, we get –1 ≤ a – 2 ≤ 1.

If a – 2 = 1 then b – 2 = c – 2 = d – 2 = 1 and a = b = c = d = 3.

If a – 2 = 0, then c – 2 = 1 and d – 2 = 2 (or c – 2 = 2 and d – 2 = 1). It follows that c ⋅ d = 12, a = 2,

that is b = 6.

If a – 2 = –1, then a = 1 and b + c + d – 2 = b = cd. Hence c + d = 2, implying c = d = 1 and b = 1.

The solutions are (a, b, c, d) = (3, 3, 3, 3); (1, 1, 1, 1); (2, 6, 3, 4); (6, 2, 3, 4); (2, 6, 4, 3); (6, 2, 4, 3);

(3, 4, 2, 6); (3, 4, 6, 2); (4, 3, 2, 6); (4, 3, 6, 2).

Solution to Solution to Solution to Solution to Problem Problem Problem Problem 99999999

As n is even, we have an – b

n = (a

2 – b

2)(a

n–2 – a

n–4b

2 + … + b

n–2).

Since a + b is a divisor of a2 – b

2, it follows that a + b is a divisor of a

n – b

n. In turn, a + b divides

2an = (a

n + b

n) + (a

n – b

n), and 2b

n = (a

n + b

n) – (a

n – b

n). But a and b are coprime numbers, and so

g.c.d.(2an, 2b

n) = 2. Therefore a + b is a divisor of 2, hence a = b = 1.

Solution to Solution to Solution to Solution to Problem Problem Problem Problem 101010100000

Suppose, by way of contradiction, that u = 23 x− and v = 3 3xa− are rational numbers. It

follows that x2 = 3 – u

2 and x

3 = a – v

3, that is a – v

3 = ±(3 – u

2) 23 u− . It follows that 23 u− = q has

to be rational, and 3 = u2 + q

2, both u and q being rationals.

Let m, n, p be integers with g.c.d.(m, n, p) = 1, such that u = p

m and v =

p

n. Then 3p

2 = m

2 + n

2,

that is 3 is a divisor of m2 + n

2. It is easy to see that 3 has to be a divisor of both m and n. Furthermore

9 is a divisor of 3p2, implying that 3 divides p. Since g.c.d.(m, n, p) = 1 we get a contradiction.

Solution to Solution to Solution to Solution to Problem Problem Problem Problem 101010101111

Denote by n2 the perfect square and by a the digit that appears in the last four positions. It easily

follows that a is one of the numbers 0, 1, 4, 5, 6 or 9. It follows n2 ≡ a ⋅ 1111(mod 10

4) and

consequently n2 = a ⋅ 1111(mod 16).

When a = 0 we are done. Suppose that a is 1, 5 or 9. Since n2 ≡ 0 or 1 or 4 (mod 8) and 1111 ≡

≡ 7 (mod 8), we obtain 1 ⋅ 1111 ≡ 7 (mod 8), 5 ⋅ 1111 ≡ 3 (mod 8) and 9 ⋅ 1111 = 7 (mod 8). Thus the

congruence n2 ≡ a ⋅ 1111 (mod 16) cannot hold.

Suppose a is 4 or 6. As 1111 ≡ 7 (mod 16), 4 ⋅ 1111 ≡ 12 (mod 16) and 6 ⋅ 1111 ≡ 10 (mod 16). We

conclude that in this case the congruence n2 ≡ a ⋅ 1111 (mod 16) cannot hold either.

Page 70: Carte Olimpiada_balcanica-2010

70707070

Solution to Solution to Solution to Solution to Problem Problem Problem Problem 111100002222

Let d = g.c.d.(x, y) and x = da, y = db, where (a, b) = 1. It is easy to see that a and b are both odd

numbers and an + b

n = 2

k for some integer k.

Suppose that n is even. As a2 ≡ b2

≡ 1 modulo 8 we have also an ≡ b

n ≡ 1 modulo 8. As 2

k = a

n + b

n

≡ 2 (mod 8), we conclude k = 1 and a = b = 1, thus x = y = d.

The equation becomes xn = 2

m–1. It has an integer solution if and only if n is a divisor of m – 1 and

x = y = n

m 1

2

.

Consider the case when n is odd. From the decomposition an + b

n = (a + b)(a

n–1 – a

n–2b + … + b

n–1),

we easily get a + b = 2k = a

n + b

n. In this case a = b = 1, and the proof goes on the lines of the

previous case.

To conclude, the given equation has solutions if and only if n

m 1− is an integer and in that case

x = y = 2p.

Solution to Solution to Solution to Solution to Problem Problem Problem Problem 101010103333

The sum of all divisors of n is given by the formula

(1 + p + p2 + … + p

a)(1 + q + q

2 + … +q

b).

The number n has (a + 1)(b + 1) positive divisors and their arithmetic mean is

)1)(1(

)...1)(...1( 22

++

++++++++=

ba

qqqpppm

ba

.

If p and q are both odd numbers, we can take a = p and b = q and it is easy to see that m is an

integer.

If p = 2 and q is odd, one can choose again b = q and a + 1 = 1 + q2 + … + q

q–1. Then m = 1 + 2 +

+ 22 + … + 2

a, and it is an integer.

For p odd and q = 2, we choose a = p and b = p2 + … + p

p–1, concluding the proof.

Solution to Solution to Solution to Solution to Problem Problem Problem Problem 101010104444

Denote S = 431

42

41 ... nnn +++ and A = {n1, n2, …, n31}.

Firstly, observe that 2 ∈ A, otherwise all numbers ni, i = 31,1 are odd and consequently S must be

odd, a contradiction.

Then, 3 ∈ A, else ni ≡ –1(mod 3) and 4in ≡ 1(mod 3) for all i = 31,1 . It follows that S ≡ 31 ≡ 1 (mod 3),

a contradiction.

Finally, we prove that 5 ∈ A. Indeed, if else, then ni ≡ ±1 (mod 5) or ni ∈ ±2 (mod 5) for all i = 31,1 .

Consequently, 2in ≡ ±1 (mod 5) and 4

in ≡ 1 (mod 5) for all i = 31,1 . Thus, S ≡ 31 ≡ 1 (mod 5), a

contradiction.

Solution to Solution to Solution to Solution to Problem Problem Problem Problem 111105050505

Assume that such numbers exist. By squaring,

(1) 2n + 1 + 2422 2 +<++<+ nxyyxnn .

Page 71: Carte Olimpiada_balcanica-2010

71717171

Since 4n + 1 < x + y + xy2 ≤ 2(x + y), we obtain x + y > 2n +2

1. Numbers x and y are integers,

so

x + y ≥ 2n + 1.

Set a = x – y – (2n + 1) ≥ 0, where a is an integer. The second inequality from (1) gives xy2 <

< 2n + 1 – a, hence 4xy < (2n + 1 – a)2. Numbers 4xy and 2n + 1 – a are also integers, therefore 4xy ≤

≤ (2n + 1 – a)2 – 1 and then 1)12(2 2 −−+≤ anxy . From (1) we have

+<+ ann22 1)12(2 2 −−++≤ anaxy ,

hence

(2) 1)12(1)12( 22 −−+≤−−+ anan .

As x + y < 4n + 2, then a = x + y – (2n + 1) ≤ 2n and so a < 1)12( 2 −+n . By squaring both sides

of the relation (2) we obtain

(2n + 1)2 – 1 + a

2 – 1)12(2 2 −+na < (2n + 1 – a)

2 – 1 ⇔

⇔ – 1)12(2 2 −+na < –2a(2n + 1),

a contradiction.

Solution to Solution to Solution to Solution to Problem Problem Problem Problem 111106060606

Number a has an odd number of digits, hence 102k ≤ a < 10

2k+1 for some integer k > 0. It suffices to

prove that a = 102k.

Firstly, observe that 104k ≤ a

2 < 10

4k+2. Number a

2 has also an odd number of digits, hence 10

2k ≤

≤ a < 2

12

10+k

. Next, 108k ≤ a

4 < 10

8k+2 and consequently 10

2k ≤ a < 4

12

10+k

. Inducting on n we obtain

102k ≤ a < n

k2

12

10+

for all n > 0.

Assume by contradiction that a ≥ 102k + 1. Then n

k2

12

10+

> 102k + 1 ⇔ 111010 2

1

2 >

−+ nk ⇔

n

kkn

2

222

1

10

1110

10

1110

+>⇔+> , for all n > 0.

On the other hand, using Bernoulli inequality we find that

k

nn

k 2

2

2 10

21

10

11 +≥

+ for all n > 0.

For sufficiently large n we have k

n

210

21+ > 10, a contradiction.

Page 72: Carte Olimpiada_balcanica-2010

72727272

Solution to Solution to Solution to Solution to Problem Problem Problem Problem 111107070707

Rearrange the numbers a1, a2, …, an in ascending order: b1 < b2 < … < bn. Obviously, k ≤ bk, and

substituting bk with k, the left-hand side term increases. Furthermore, by the Rearrangements

inequality we infer that the maximum value of the left-hand side term is

1...

1

21 n

nn++

−+ .

On the other side, the right-hand side term is greater than or equal to

4

)1(

2

...21 +=

+++ nnn.

We have

∑=

+−=++

−+

n

k k

knn

nn 1

1

1...

1

21 =

= ∑∑∑+

===

+=++=−+1

221

1)1(

1)1(1

1)1(

n

k

n

k

n

k kn

knn

kn .

For n > 6 we prove by induction on n that

∑+

=

≥1

2

1

4

n

k k

n,

which implies that the given equality cannot hold. Indeed, for n = 7 we have 4

7 = 1.75 ≥ ++

3

1

2

1 … +

+ 8

1= 1.51…

If the inequality holds for n > 7 then it is true for n + 1, as 1

1

4

1

+≥n

.

We are left with the cases when n = 2, 3, 4, 5, 6. Clearly, the case case n = 2 is impossible.

For n = 3 we have the numbers a1 = 1, a2 = 2 and a3 = 3, so n = 3 is a solution.

If n = 4, then

a1 + a2 + a3 + a4 = 131

4

2

3

3

2

4

12

43212

4321

<

+++≤

+++aaaa

,

so a1 + a2 + a3 + a4 ≤ 12. By inspection, all the cases: {a1, a2, a3, a4} = {1, 2, 3, 4}, {1, 2, 3, 5}, {1, 2,

4, 5} and {1, 2, 3, 6} fail to satisfy the required relation.

If n = 5, then

a1 + a2 + a3 + a4 + a5 = 4.171

5

2

4

3

3

4

2

5

12

543212

54321

<

++++≤

++++aaaaa

,

so a1 + a2 + a3 + a4 + a5 ≤ 17. We study the cases {a1, a2, a3, a4, a5} = {1, 2, 3, 4, 5}, {1, 2, 3, 4, 6} and

{1, 2, 3, 5, 6} with no success (for an easy argument, observe that 5 must be a5 and so on).

Finally, for n = 6 we obtain similarly a1 + a2 + … + a6 ≤ 22, thus {a1, a2, a3, a4, a5, a6} can be {1, 2,

3, 4, 5, 6} or {1, 2, 3, 4, 5, 7}. The last case fails immediately because of 7, and the same outcame is

for the first one.

Therefore n = 3.

Page 73: Carte Olimpiada_balcanica-2010

73737373

Solution to Solution to Solution to Solution to Problem Problem Problem Problem 111108080808

Clearly one of the primes p, q or r is equal to 2. If r = 2 then pn + q

n = 4, false, so assume that

p > q = 2.

Consider the case when n > 1 is odd; we have 213221 )2...22)(2( rpppp nnnn =+−+−+ −−−− .

Notice that ...))(2(22...22 42113221 ++−+=+−+− −−−−−−− nnnnnnn pppppp > 1 and p + 2 > 1 hence

both factors are equal to r. This rewrites as pn + 2

n = (p + 2)

2 = p

2 + 4p + 4, which is false for n ≥ 3.

Consider the case when n > 1 is even and let n = 2m. It follows that pm = a

2 – b

2, 2

m = 2ab and r =

a2 + b

2, for some integers a, b with (a, b) = 1. Therefore a and b are powers of 2, so b = 1 and a = 2

m–1.

This implies pm = 4

m–1 – 1 < 4

m, so p must be equal to 3. The equality 3

m = 4

m–1 – 1 fails for m = 1 and

also for m ≥ 2, as 4m–1

> 3m + 1, by induction.

Consequently n = 1 – take for example p = 23, q = 2 and r = 5.

Solution to Solution to Solution to Solution to Problem Problem Problem Problem 111109090909

The sum of the elements of the set A is S = na2 – n(n + 1)b + 2

6

)12)(1(b

nnn ++. Assume that n has

a prime divisor p > 3. Then p | S and p | (a + b)2 + (a + 2b)2 + … + (a + pb)

2 = pa

2 + p(p + 1)b +

+ 2

6

)12)(1(b

ppp ++, a contradiction. It remains n = 2

k3l, for some integers k, l. Suppose that k > 1.

Then 2 | S and so all elements of A must be odd. Taking the subset given by any pair we reach the con-

tradiction. Finally, suppose that l > 1, so 3 | S. If one of the numbers a + b, a + 2b, a + 3b is divisible

by 3, then we have a contradiction; if not, then 3 | b. Then 3 | (a + b)2 + (a + 2b)

2 + (a + 3b)

2 = 3a

2 +

+ 12ab + 14b2, again a contradiction.

We are left with n = 6, which satisfies the claim: the set A = {4, 9, 16, 25, 36, 49} is isolated,

because the sum of its elements is a prime number (139).

SSSSolution to olution to olution to olution to Problem Problem Problem Problem 111111110000

The required number is 505.

At start, note that the remainder of n when divided by 4 is odd, hence n is odd.

Furthermore, observe that the quotient of n when divided to a square less then 2

n is greater than or

equal to 2. On the other side, the quotient at a division by an odd square cannot equal 3, as the

remainder would be even. Consequently, there are no positive integers k so that 3 ≤ 2)12( −k

n < 4, in

other words there is no k ∈ N with

3)12(

4

2 nk

n≤−< .

Let m ∈ N* so that (2m – 1)

2 ≤

34

nn< < (2m + 1)

2. Then (2m + 1)

2 – (2m – 1)

2 >

43

nn− , hence

8m > 12

n. It follows that

Page 74: Carte Olimpiada_balcanica-2010

74747474

(2m – 1)2 ≤

4

n < 24 ⋅ m,

so m ∈ {1, 2, …, 6}. Since n < 96 ⋅ m ≤ 576, then the odd squares less then 2

n < 288 are 9, 25 …, 225.

Recall that the quotients at the division by 9, 25, …, 225 are even, so the quotients at the division by

225 and 169 are both 2 (else 4 ⋅ 169 > 576).

Thus n = 450 + a = 338 + b with 0 < a < 225, 0 < b ≤ 137 and a, b are odd, so n ≤ 338 + 137 = 505.

For n = 505 one can easy check the claim.

Solution to Solution to Solution to Solution to Problem Problem Problem Problem 111111111111

At first, note that if r = 1, then the new number is obviously rational.

Furthermore, if x is a simple rational number (that is with no period), then again the new number is

rational.

Now assume that x has a period and let p ≥ 1 the number of digits of the period.

Let m > 0 be the rank of the last decimal which do not belong to the period, thus

x = 0,a1a2…am(am+1…am+p).

Let l = k + ir > m, i ∈ N. Then

x = 0,a1a2…al(al+1…al+r…al+pr).

The number obtained by canceling the decimals ak, ak+r, ak+2r, ak+3r, … will have the period

al…al+r–1al+r+1… al+pr–1,

therefore it will be a rational number.

SoluSoluSoluSolution to tion to tion to tion to Problem Problem Problem Problem 111111112222

Suppose that n ≥ p. Then n8 – n

2 = n

2(n

3 – 1)(n

3 + 1) ≥ n

2n

2(n

3 + 1) = n

7 + n

4 ≥ p

7 + p

5 > p

5 + p

2, a

contradiction. Therefore n < p. As p is prime and p2 | n2

(n3 – 1)(n

3 + 1), it follows that p

2 | n3

– 1 or

p2 | n3

+ 1, so p2 ≤ n

3 + 1. Next we have n

2(n

3 – 1)(n

3 + 1) = p

5 + p

2 ≤ 53 )1( +n + (n

3 + 1), hence

(1) 11)1()1( 3332 +++≤− nnnn ,

On the other hand n3 + 1 < 22 )1( −n , since this rewrites as n

2 ≥ n + 2. The relation (1) yields

n2(n

3 – 1) ≤ (n

3 + 1)(n

3 – 1) + 1 = n

5 – n

3 + n

2, then n

3 ≤ 2n

2. Consequently n = 2 and p

2 ≤ n3

+ 1 = 9,

thus p = 3.

Solution to Solution to Solution to Solution to Problem Problem Problem Problem 111111113333

The numbers are 18 and 27.

Let k be the number of digits of n in decimal representation. Notice that:

(1) n = p ⋅ s(n), where p is prime so that any prime divisor of s(n) is greater than or equal to p;

(2) s(n)2 ≥ n, so 10

k–1 ≤ n ≤ s(n)

2 ≤ (9k)

2, hence k ≤ 4.

We study the following cases:

a) If k = 4, then n = abcd , n ≤ s(n)2 ≤ 36

2 = 1296, so a = 1. Then s(n) ≤ 28, thus n ≤ 28

2 < 1000,

false.

b) If k ≤ 3, then abc , so 9(11 ⋅ a + b) = (p – 1)(a + b + c).

Page 75: Carte Olimpiada_balcanica-2010

75757575

If 9 divides p – 1, since p < a + b + c = 27 we get p = 19. Next 9a = b + 2c, hence a ≤ 3. As a + b +

+ c ≥ 23 – see (1) – we have no solution.

If 9 do not divide p – 1, from 3 | a + b + c and (1) we get p = 2 or p = 3.

For p = 3 we have n = 3(a + b + c), so a = 0 and 10 ⋅ b + c = 3(b + c). Consequently, 7b = 2c and

n = 27.

For p = 2 we get n = 2(a + b + c), so a = 0 and 8b = c, hence n = 18.

Solution to Solution to Solution to Solution to Problem Problem Problem Problem 111114141414

Without loss of generality, assume ak > bk, k = 4,1 . Then a1 + a2 = a3 + a4 and b1 + b2 = b3 + b4. We

analyze 2 cases:

i) a1 + a2 = a3 + a4 and a2 + b1 = a4 + b3. Subtracting we get |a2 – b2| = |a4 – b4|, |a1 – b1| = |a3 – b3| and the claim is obvious.

ii) a1 + b2 = a4 + b3 and a2 + b1 = a3 + b4. By subtraction we obtain |a2 – b2| = |a3 – b3|, |a1 – b1| = = |a4 – b4|, as needed.

Solution to Solution to Solution to Solution to Problem Problem Problem Problem 111111115555

Assume by contradiction that the claim holds and let b ≤ a. The number r(b) has at most as many

digits as b, so r(b) < 10b ≤ 10a. It follows that

(2a)2 < 4a

2 + 10a < (2a+ 3)

2,

hence 4a2 + r(b) = (2a + 1)

2 or (2a + 2)

2, thus r(b) = 4a + 1 or 8a + 4. Notice that r(b) > a ≥ b,

implying that a and b have the same number of digits. Then, as above, we get r(a) ∈ {4b + 1, 8b + 4}.

We analyze 3 cases:

1. r(a) = 4b + 1 and r(b) = 4a + 1. Subtracting we get (r(a) – a) + (r(b) – b) = 3(b – a) + 2, which is

false since 9 divides r(n) – n for any positive integer n.

2. r(a) = 8b + 4 and r(b) = 4a + 1 (the same reasoning is to be applied for r(b) = 8a + 4 and r(a) =

= 4b + 1). Subtracting we obtain (r(a) – a) + (r(b) – b) = 7b + 3a + 3, so 3 divides b. Then 3 divides

also r(b) = 4a + 1, so a and r(a) have the remainder 2 when divided at 3. This leads to a contradiction

with r(a) = (8b + 3) + 1.

3. r(a) = 8b + 4 and r(b) = 8a + 4. Both r(a) and r(b) have the last digit even, so at least 2. Then a

and b have the first digit greater than or equal to 2, so 8a + 4 and 8b + 4 have more digits than a and b.

It follows that r(a) < 8b + 4 and r(b) < 8a + 4, a contradiction.

Solution to Solution to Solution to Solution to Problem Problem Problem Problem 111116161616

Let m = ][ n . Since n ≥ 4, then m ≥ 2 is an integer. We have m2 ≤ n < (m + 1)

2, so

m2 ≤ n ≤ m

2 + 2m.

Set n = m2 + k, k = 0, 1, 2, …, 2m. From m – 1 | m

2 + k – 1 we get m – 1 | k. On the other hand

k < 2(m + 1), thus k = 0 or k = m + 1.

If k = 0, from m – 1 | m2 + 1 follows m – 1 | 2, so m = 2 or m = 3, hence n = 4 or n = 9.

If k = m + 1, then m – 1 | m2 + m + 2 = m

2 – 1 + m – 1 + 4, so m – 1 | 4. We obtain m = 2, 3 or 5,

hence n = 7, 13 or 31.

Therefore, n ∈ {4, 7, 9, 13, 31}.

Page 76: Carte Olimpiada_balcanica-2010

76767676

Solution to Solution to Solution to Solution to Problem Problem Problem Problem 111117171717

At first, notice that (x – 1)(x – 2) ≥ 0 for all x ∈ N, (y – 1)(y – 3) ≥ 0 if y ∈ N \ {2} and (z – 1)(z – 4) ≥

≥ 0 when z ∈ N \ {2, 3}. In other words, if y ≠ 2 and z ∉ {2, 3}, then x2 + 2 ≥ 2x, y

2 + 3 ≥ 3y and

z2 + 4 ≥ 4z. Multiplying the above inequalities yields (x

2 + 2)(y

2 + 3)(z

2 + 4) ≥ 60xyz, so in all three

inequalities the equality must occur. Until now we have the solutions:

(x, y, z) = (1, 1, 1), (1, 1, 4), (1, 3, 1), (2, 1, 1), (2, 3, 1), (2, 1, 4), (1, 3, 4), (2, 3, 4).

We claim the there are no more solutions. For this, we will show that if z = 2 or z = 3 or y = 2, there

are no integers satisfying the given equation.

The quadratic residues modulo 5 are 0, 1, 4, so 5 do not divide neither x2 + 2 nor y

2 + 3. Since 5

divides 60xyz, it follows that 5 divides z2 + 4, hence z ∈ {5k ± 1 | k ∈ Z}. As a consequence, z ≠ 2 and

z ≠ 3.

If y = 2, the equation rewrites as 120xz = 7(x2 + 2)(z

2 + 4), from which we may notice that 8 divides

(x2 + 2)(z

2 + 4). If x, z are even integers, then x

2 + 2 is even and z

2 + 4 is divisible by 4, but 4 P x2

+ 2

and 16 P z2 + 4, so the power of 2 in the right-hand side is at most 4, while in the left hand-side is at

least 5, a contradiction. If only one of the numbers x and z is even, the contradiction is reached

similarly. Hence y ≠ 2 and the only solutions of the equation are the ones previously obtained.

Solution to Solution to Solution to Solution to Problem Problem Problem Problem 111118181818

Any integer which can be represented as described in the problem will be called good.

Setting b = c = 1 yields [a, b] + [b, c] + [c, a] = a + 1 + a = 2a + 1, hence any odd integer is good.

Notice that [2x, 2y] = 2 ⋅ [x, y]. Therefore, if n can be represented as [a, b] + [b, c] + [c, a], then 2n

writes as [2a, 2b] + [2b, 2c] + [2c, 2a] = 2([a, b] + [b, c] + [c, a]), thus all integers with are not powers

of 2 are good.

We claim that all numbers of the form 2k, k ∈ N are not good. For k = 0 and k = 1 this is obvious, as

[a, b] + [b, c] + [c, a] ≥ 1 + 1 + 1 = 3. If k ≥ 2, suppose by contradiction that there exist a, b, c as

needed. Let a = 2A ⋅ a1, b = 2

B ⋅ b1, c = 2

C ⋅ c1, where a1, b1, c1 are odd. Without loss of generality,

assume that A ≥ B ≥ C. Then 2k = [a, b] + [b, c] + [c, a] = 2

A([al, b1] + [a1, c1]) + 2

B ⋅ [b1, c1]. Dividing

by 2B, k > B yields 2

k–B = 2

A–B ⋅ ([al, b1] + [a1, c1]) + [b1, c1]. But [al, b1] + [a1, c1] is even and [b1, c1] is

odd, contradiction.

Solution to Solution to Solution to Solution to Problem Problem Problem Problem 111119191919

Suppose that p2 P a + b. It suffices to prove that p

3 | a3

+ b3. Indeed, if p

2 | (a + b)

3 – 3ab(a + b), we

have p | 3ab. As p ≠ 3 is prime, it follows that p | a or p | b. Since p | a + b, we get p | a and p | b. As a

consequence, p3 | a

3 and p

3 | b3

, implying p3 | a

3 + b

3.

Solution to Solution to Solution to Solution to Problem Problem Problem Problem 121212120000

Let n ≥ 1 and let 10k, k ∈ N. Consider all the remainders of the numbers 10

k at the division by n.

Since there are only finitely many residues, there exists a = 0, 1, …, n – 1 so that 10m ≡ a (mod n) for

infinitely many values of m ∈ N. Among them select only n, namely nmmm10...,,10,10 21 , with m1 >

> m2 > … > mn. The number A = nmmm10...1010 21 +++ has n digits equal to 1 and the rest equal to 0,

has the sum of all digits is n. Moreover, A ≡ na ≡ 0 (mod n), so n | A, which concludes the proof.

Page 77: Carte Olimpiada_balcanica-2010

77777777

Solution toSolution toSolution toSolution to Problem Problem Problem Problem 121212121111

Consider An–1 = an – an–1. Since an ≤ n and an–1 ≥ 1, we have An–1 ≤ n – 1.

If An–1 = 0, that is an–1 = an, then a1 + a2 + … + an–2 is even and the claim reduces to the case of n – 2

numbers.

If An–1 > 0, then a1 + a2 + … + an–2 is even and the claim reduces to the case of n – 1 numbers.

Solution to Solution to Solution to Solution to Problem Problem Problem Problem 121212122222

Notice that a1 = n, as 1 divides any aj, and aj ≤ n, for any j = 1, 2, …, n.

Consider an array with rows corresponding to 1, 2…., n and columns corresponding to the numbers

a1, a2, …, an. Define the entries as follows, on the position (i, aj) put 1 if i divides aj and 0 otherwise.

Now observe that the sum of the numbers placed of the ith row is equal to ai, as we see the entries 1 for

each multiple of i in the sequence a1, a2, …, an. Hence the sum of all the entries in the array is ∑ ja .

On the other hand, on the thja column we get and entry 1 as long as we count the divisors of aj, so

the sum of the numbers on the jth columns is the number of divisors of aj. This implies that the sum of

all the entries in the array is the sum of all divisors of the numbers aj.

Using this double-counting, together with the obvious fact that the number of divisor of a is less

than a – unless a is 1 or 2, show that n = 1 or n = 2.

Alternative solution. Recall that a1 = n and ai ≤ n, for all i = 1, 2, …, n.

Assume that n > 3. As an–1 ≥ 1, there exists a multiple of n – 1, where n – 1 ≥ 1, in the given

sequence; let ak, k > 1 be such a multiple. The condition ai ≤ n shows that ak = n – 1, in other words

there are n – 1 multiples of k in the sequence. As n and n – 1 are coprime, k does not divide a1 = n, so k

divides a2, …, an. But k ≥ 2 and k | an, therefore an > 1. Thus n must occur at least twice in the

sequence, so, beside a1 we have aj = n, j > 1. Hence k | n, a contradiction. As before, n = 1 or n = 2 are

the only possible values.

Solution to Solution to Solution to Solution to Problem Problem Problem Problem 121212123333

Let [an + b] = 2xn, for all integers n > 0. Then

(1) 2xn ≤ an + b < 2xn + 1, (2) 2xn+1 ≤ a(n + 1) + b < 2xn+1 + 1.

Subtracting (1) from (2) we get 2(xn+1 – xn) – 1 < a < 2(xn+1 – xn) + 1, for all n > 0. As 2(xn+1 – xn) – 1

is an odd integer, it follows that all numbers 2(xn+1 – xn) – 1 must be equal, as otherwise a belongs to

two open intervals of lengths 2 having the left margins at a difference of at least 2, which is

impossible. Hence 2(xn+1 – xn) – 1 = 2s – 1, s ∈ Z, so xn+1 – xn = s and then xn = ns + p, p ∈ Z, ∀n > 0.

Plugging in (1) we get 2p – b ≤ (a – 2s)n < 2p – b + 1, ∀n > 0, so a = 2s, since otherwise the set of

the positive integers will have an upper margin. Observe that s is an integer, so a is an even integer, as

needed.

Solution to Solution to Solution to Solution to Problem Problem Problem Problem 111124242424

It is easy to observe that p is odd and p ≠ q. in other words p ≥ 3 and (p, q) = 1.

If q = 2, then 2p+1

= 7 + p2. The only solution is p = 3, as 2

n+1 > 7 + n

2, for all n ≥ 4.

For q ≥ 3, by Little Fermat’s Theorem we get p | 2q – 7 and q | p + 7. Set p + 7 = kq, k ∈ N*.

If 2q – 7 ≤ 0, we have q = 3 and p | –1, false.

Page 78: Carte Olimpiada_balcanica-2010

78787878

If 2q – 7 > 0, then 2q – 7 ≥ p, so 2q ≥ p + 7 ≥ kq, therefore k = 1 or k = 2. For k = 1 we obtain

p + 7 = q, so p | 2p + 7. This implies p = 7 and then q = 14, false. Hence k = 2 and p + 7 = 2q. Suppose

p > q; as p, q ≥ 3 we get qp ≥ qp and then 7 = 2q

p – p

q ≥ qp ≥ 3

3 = 27, a contradiction. Thus q > p and

then p + 7 = 2q > 2p, which yields p = 3 or p = 5. For p = 3 we have q = 5, while p = 5 gives q | 12,

with no solution.

To conclude, the solutions are (p, q) = (3, 2), (3, 5).

Solution to Solution to Solution to Solution to Problem Problem Problem Problem 111125252525

The solutions are (k, k2) and (k

2, k), with k > 1.

We have mn – 1 | (n3 – 1)m – n

2(mn – 1) = n

2 – m. On the other hand, mn – 1 | m(n

2 – m) – (mn –

1)n = n – m2.

If n > m2, then mn – 1 ≤ n – m

2 ≤ n – 1, so mn ≤ n, false.

If n = m2, then obviously m

3 – 1 | m6

– 1, so all pairs (m, m2), m > 1 are solutions.

If n < m2, from mn – 1 ≤ n

3 – 1 we derive that

2nmn ≤< . Then mn – 1 ≤ m

2 – n < m

2 – 1, so

n < m. If n2 – m > 0, we obtain mn – 1 ≤ n

2 – m < n

2 – 1, so m < n, a contradiction. Hence n = m

2,

which holds, since m3 – 1 | m3

– 1, so all pairs (n2, n), n > 1 are also solutions.

Solution to Solution to Solution to Solution to Problem Problem Problem Problem 111126262626

Since is not divisible by 2 nor by 5, the number a2009 has a multiple M with all digits equal to 1.

Let m > 0 be the number of digits of M, that is: M = ���times

1...11m

. As a2009+m = a2009 ⋅ 10m + 3M, we get

a2009 | a2009+m, as needed.

Solution to Solution to Solution to Solution to Problem Problem Problem Problem 111127272727

Considering the remainders modulo 3 we get d = 0, otherwise 1 = 1 + (–1)c, false. The equation

rewrites as 7a = 4

b + 5

c + 1.

Assuming b ≠ 0, the left-hand side term is odd while the right hand-side is even, so b = 0 and then

7a = 5

c + 2.

It is obvious that c ≥ 1. Taking the remainders modulo 5 we obtain 2a = 2, hence a = 1 + 4k and

7(49)2k = 5

c + 2. If k ≥ 1 then c ≥ 2. Modulo 25 we get 7(–1)

2k = 0 + 2, false. It follows that k = 0, then

a = 1 and c = 1.

Therefore a = 1, b = 0, c = 1 and d = 0.

Solution to Solution to Solution to Solution to Problem Problem Problem Problem 111128282828

Suppose n and n + 1 are saturated. Then the numbers 4n(n + 1) and 4n(n + 1) + 1 = (2n + 1)2 are

also saturated, q.e.d.

Page 79: Carte Olimpiada_balcanica-2010

79797979

Chapter IV

COMBINATORICS

Solution to Solution to Solution to Solution to Problem Problem Problem Problem 111129292929

a) If n = 2k, there are no such symmetrical tilings (otherwise the k and k + 1 tiles must have the

same colour).

If n = 2k + 1, the problem is to count the possible tilings for k + 1 squares. There are 4 ⋅ ����� 3...33 ⋅⋅⋅ =

= 4 ⋅ 3k such tilings.

b) There are 4 ⋅ 3 ⋅ ����� 2...22 ⋅⋅⋅ = 4 ⋅ 3 ⋅ 2n–2 tilings.

Solution to Solution to Solution to Solution to Problem Problem Problem Problem 131313130000

The first step give rise to one black square of area 9

1

3

12

=

. After the second step we obtain eight

more squares of side 9

1, the black region increasing by

29

8. In the same manner, the third step

increases the black area by 82 = 64 black squares, each of area

27

1, that is at this stage the black area

becomes

3

2

2 9

8

9

8

9

1++ .

We conclude that after 1000 steps, the area of the black region is

=

++

++=++++9992

1000

999

3

2

2 9

8...

9

8

9

81

9

1

9

8...

9

8

9

8

9

1

1000

1000

9

81

9

81

9

81

9

1

−=

−⋅= .

It is left to prove that the last number is greater than 0.001. This follows using a binomial

expansion evaluation, namely

1000642

9991000

8

1

2

1000

8

11

8

92

10001000

>⋅

⋅=⋅

>

+=

.

Consequently, 9999,01000

11

9

81

1000

=−>

− .

Page 80: Carte Olimpiada_balcanica-2010

80808080

Solution to Solution to Solution to Solution to Problem Problem Problem Problem 131313131111

Let us consider the general case, that is to consider the number an of equilateral triangles formed by

division in n segments. We shall find a recurrence relation.

Consider an equilateral triangle with sides partitioned into n + 1 equal segments and draw the n

parallels to each side of the given triangle. We will count all triangles with at least one vertex on BC;

the remaining ones are triangles counted in an.

Consider first the triangles that have two vertices on BC. When choosing two division points on

BC, say M and N with M ∈ (BN), one counts exactly one triangle, namely that one obtained by

drawing parallels from M, N to AB, AC respectively. Hence we add 2

)1)(2( ++ nn new triangles.

Consider the triangles with only one vertex on (BC). For each of the n division points we count one

unit triangle. Except for the two points closer to B and C respectively, we count n – 2 triangles of side

2, and so on. Therefore,

+−+−++++

+=+ )4()2(2

)1)(2(1 nnn

nnaa nn …

Changing n in n + 1, we get

=−+−+++++

+= ++ )3()1()1(2

)2)(3(12 nnn

nnaa nn …

Summing up, we obtain

5

)53)(2(

2

)2)(1(

2

)2)(3(

2

)1)(2(2

+++=

+++

+++

+++=+

nna

nnnnnnaa nnn .

It follows that

315315...2371452

)583(10068810 =+==+=+=

+⋅+= aaaaa .

Therefore, the number of triangles is 315.

Solution to Solution to Solution to Solution to Problem Problem Problem Problem 131313132222

Choose an arbitrary point A in the plane. Points located in the plane at a rational distance from A

are colored in red, while the others are colored in blue. Consider an arbitrary segment PQ. We may

assume that AP < AQ; if not, take instead P another point of the line segment (PQ).

Recall that between two real numbers one can find a rational number q and an irrational number r.

The circles centered at A and having the radii q and r intersect the segment PQ at the points M and N

respectively. It is obvious that M is colored in red and N in blue, so the claim is proved.

Solution to Solution to Solution to Solution to Problem Problem Problem Problem 131313133333

Color the 8 vertices of the cube in black or white such that the 4 vertices of the 2 regular

tetrahedrons have the same color; notice that the 3 vertices adjacent to a vertex have its opposite color.

Therefore, each movement increases the sum of the numbers assigned to the vertices sharing the same

color by 3. Consider the cases:

1) MN is a diagonal of a face of the cube. Then M and N have the same color, say black. Assume

by contradiction that there is a sequence of movements after which the same number n is assigned to

all vertices. Let k1 and k2 be the number of white, respectively black vertices that were selected to

Page 81: Carte Olimpiada_balcanica-2010

81818181

perform the movements. Then

4n = 3k1 + 2 = 3k2,

a contradiction.

2) MN is a diagonal of the cube. Selecting the vertices M, then N, and performing these 2

movements, the number 1 will be assigned to all the vertices, as needed.

3) MN is a side of the cube. The same outcome as in the previous case will occur after 2 movements

when selecting the diagonally opposite vertices of M and N.

This provides us with a full solution.

Solution to Solution to Solution to Solution to Problem Problem Problem Problem 134134134134

The main idea is to observe that two consecutive rows have exactly 4 equal elements, namely those

lying on the columns 1, 3, 5, 7 or 2, 4, 6, 8. Moreover, on the other 4 columns the elements are

different. Wlog, assume that rows 1 and 2 are equal with respect to the columns 1, 3, 5, 7 and different

on the column 2, 4, 6, 8; we call these rows odd equal. If rows 2 and 3 are also odd equal, then rows 1

and 3 are equal, as needed. If not, then rows 2 and 3 are even equal. Now consider the rows 3 and 4;

we are done if the rows are even equal, so assume that they are odd equal. Finally, if rows 4 and 5 are

odd equal, then rows 3 and 5 are equal, and if rows 4 and 5 are even equal, then rows 1 and 5 are

equal. This concludes the proof.

Solution to Solution to Solution to Solution to Problem Problem Problem Problem 111135353535

Let n be the number of players in the tournament. The total numbers of matches is n(n – 1), hence

each player end up with n – 1 points.

a) Assume by contradiction that each player has a different number of draws. As a draw gives 0.5

points, it follows that each player has an odd number of draws. Since the possible cases are: 0, 2, 4, …,

2(n – 1), we infer that each of these numbers is assigned to each of the players. Consider A the player

with 0 draws and B the player with 2(n – 1) draws. Each player has played 2(n – 1) matches, hence B

obtained a draw in each match played. The match A – B thus ended with a draw, a contradiction, since

A has no draws.

b) Suppose the contrary. Then each of the n players has 0, 1, …, n – 1 losses when playing the

white. Let X and Y be the players with 0 and n – 1 losses, respectively. The player Y has no points

when playing the white and n – 1 points, so he won all the matches with the black pieces. This implies

that the match X – Y is won by Y, so is lost by X, a contradiction, since X has 0 losses with the white

pieces.

Solution to Solution to Solution to Solution to Problem Problem Problem Problem 111136363636

Let A1, A2, …, A1000 be the vertices of the polygon. We start with two lemmas.

Lemma 1. Three of four consecutive vertices have the same color. Then after a sequence of moves

all vertices will have the color of the fourth vertex.

Proof. Let the colors be 0, 1 and 2. We have two cases:

a) 1110 → 1122 → 1002 → 2202 → 2112 → 0000.

b) 1011 → 1221 → 0000.

Lemma 2. Any 4 consecutive vertices will turn after several moves in the same color.

Proof. Form two pairs of consecutive vertices and change them in the same color – if they do not

already have it. Then follow the sequence 1122 → 1002 → 2202 → 2112 → 0000.

Page 82: Carte Olimpiada_balcanica-2010

82828282

By the second lemma, after several moves the vertices A1, A2, A3, A4 will have the same color, say

red. Likewise, A5, A6, A7, A8 will have the same color. Consider now the vertices A4, A5, A6, A7; the first

is red and the other three have the same color. By the first lemma they all will turn red – of course, we

do nothing if they were already red. We move on with this procedure until A1, A2, …, A997 turn red

(note that 997 = 4 + 3 ⋅ 332, so this requires 332 steps). Now consider the vertices A998, A999, A1000, A1;

by the second lemma they all will share the same color. If this is red, we are done. If not, say that they

are blue, and taking the vertices A997, A998, A999, A1000 we obtain – using the first lemma – all vertices to

be red, except for A1, which is blue. Now A1, A2, A3, A4 turn blue, then A5, A6, A7, A8 and so on. This

time, after 333 steps, all the 1000 vertices (1000 = 1 + 3 ⋅ 333) will be colored in blue.

Comment. Substituting colors with digits, notice that all moves: 01 → 22, 02 → 11 and 12 → 00

preserve the sum (mod 3). This means that the final color is unique and, of course, is given by the sum

of the digits assigned to the vertices of the initial configuration.

Solution to Solution to Solution to Solution to Problem Problem Problem Problem 131313137777

The number of routes between 2 cities is

2

6 = 15. By the pigeonhole principle one can find that a

company – say M – operates at least 8 routes.

There are

4

6 = 15 subsets of 4 cities and each pair of cities occurs in

2

4 = 6 such subsets. Using

again the pigeonhole principle follows that there exists a subset of 4 cities among which at least 4

routes are operated by the company M (3 ⋅ 15 < 6 ⋅ 8).

If those routes form a cycle, we are done. If else, then one can easy observe that among these 4

cities X, Y, Z, T, the routes X ↔ Y, Y ↔ Z, Z ↔ X and X ↔ T are operated by the company M (set the

notation accordingly).

The other 2 cities, say P and Q, are connected by at least 4 routes operated by M. Even if P ↔ Q is

one of them, from P or Q – say P – at least 2 routes to X, Y, Z, T are operated by M. If both

destinations are from X, Y, Z, a cycle is obtained, so assume that one of the routes is P ↔ T. In this

case we are done if the second route is P ↔ Y or P ↔ Z, hence we are left with the case P ↔ X.

From those above, the existence of a third route from P to X, Y, Z or T will provide a cycle. If not,

from Q exist 2 routes to X, Y, Z, T. The same line of reasoning shows that we have to consider that on

the route Q ↔ T is operated by M.

Any of the routes Q ↔ X, Q ↔ Y, Q ↔ Z will close a cycle. To conclude, if the route P ↔ Q is

operated by M, then any route from Q to X, Y, Z or T will provide us with the desired cycle.

Solution to Solution to Solution to Solution to Problem Problem Problem Problem 111138383838

1) A person can speak free of charge with at most 2k persons – k that he chooses and other k (at

most) that select his numbers among their free calls. Since n ≥ 2k + 2, each person will be charged

when speaking to (at least) another one.

2) Assume that all 2k + 1 persons are arranged in a circle. Each person will choose to speak free of

charge when calling any of the k persons located – consecutively – at his right. Then any person will

speak free of charge with the k persons located at his left, as all of them will choose him among their

“favorite numbers”.

Page 83: Carte Olimpiada_balcanica-2010

83838383

Solution to Solution to Solution to Solution to Problem Problem Problem Problem 111139393939

The sum of all integers is 1 + 2 + … + n2 =

2

)1( 22 +nn, so there is a row r having the sum of the

numbers assigned to its squares at least equal to 2

)1( 22 +nn. Consider the 2n – 2 numbers written on

the first and last column – except for the two numbers which belong to the row r – and observe that

their sum is at least 1 + 2 + … + (2n – 2) = (n – 1)(2n – 1).

Now we can select two “complementary parts” of these columns – in order to complete the row r to

a path – so that the sum of the numbers placed on these n – 1 squares is at least 2

)12)(1( −− nn. Since

2

)1( 2 +nn +

2

1

22

)12)(1( 23

+−+=−−

nnnnn

, to conclude we only have to notice that

2

3n + 1 =

= 2

1

2

3

+n

– if n odd – and that

2

3n + n

2 – n +1 is the smallest integer greater than

2

1

2

23

+−+ nnn

when n is even.

Solution to Solution to Solution to Solution to Problem Problem Problem Problem 141414140000

One can place 4, 6, …, 40, 42 checkers under the given conditions.

We start by noticing that n is the sum of 7 even numbers, hence n is also even. On a row one can

place at most 6 checkers, hence n ≤ 6 ⋅ 7 = 42.

The key step is to use 2k × 2k squares filled completely with checkers and 2k + 1 × 2k + 1 squares

having checkers on each unit square except for one diagonal. Notice that these types of squares

satisfies the conditions, and moreover, we can glue together several such squares within the problem

conditions.

Below we describe the disposal of n checkers for any even n between 4 and 42.

For 4, 8, 12, 16, 20, 24, 28, 32 or 36 checkers 1, 2, 3, 4, 5, 6, 7, 8 or 9 2 × 2 squares; notice that all

fit inside the 7 × 7 array!

For 6 checkers consider a 3 × 3 square – except for one diagonal; then adding 2 × 2 squares we get

the disposal of 10, 14, 18, 22, …, 38 checkers.

For 40 checkers we use a 5 × 5 and five 2 × 2 squares.

Solution to Solution to Solution to Solution to Problem Problem Problem Problem 141414141111

Let n be the required number. We claim that n = 2007.

With 7 cuts, from the given rectangular piece one can obtain an 11-sided polygon and some

triangles. From a triangle, with 8 cuts one can get an 11-sided polygon and some extra pieces,

sufficiently enough to continue the same procedure. Hence, using 7 + 8 ⋅ 250 = 2007 cuts one can

obtain the 251 requested 11-sided polygons.

Denote by k the number of pieces left at the end which are not 11-sided polygons and notice that

each has at least 3 sides. Now, observe that with each cut the number of pieces increases by 1 and total

the number of vertices increases with at most 4 – actually, with 2, 3 or 4, according to the number of

existent vertices through which the cutting line passes.

Page 84: Carte Olimpiada_balcanica-2010

84848484

Then n = k + 250 and 4n + 4 ≥ v ≥ 11 ⋅ 251 + 3k, where v is the total number of vertices of all

polygons at the end. Hence 4n + 4 ≥ 11 ⋅ 251 + 3(n – 250) = 2011 + 3n, so n ≥ 2007, as claimed.

Solution to Solution to Solution to Solution to Problem Problem Problem Problem 141414142222

Assign the number 0 to each white square and the number 1 to each black square. The claim is

achieved if we prove the existence of a 2 × 2 square with an odd sum of the 4 numbers inside.

Assume the contrary, so each sum of the 4 numbers inside a 2 × 2 square is even. Summing over all

squares we get an even number S. Notice that each square not sharing a common side with the given

array occurs 4 times in 5, the squares with only a common side occurs twice, while the 4 squares in the

corners only once. But in the four corners there are three 0’s and one 1, so the sum S is even, a

contradiction.

Remark. The given array may have a rectangular form, and the above solution requires no

alteration. However, this remark can easily lead to alternative solutions using induction. Here is a

sketch: choose a row with 0 and 1 at endpoints and call it the first row. Suppose that the number below

0 is also 0; arguing by contradiction, we notice that all “doubletons” formed vertically from the first

two rows have equal numbers inside, so the second row – which starts with 0 – ends with 1. Deleting

the first row of the given rectangular array, the claim is reached by induction. The same line of

reasoning is applied to the case when below 0 the number is 1.

Solution to Solution to Solution to Solution to Problem Problem Problem Problem 141414143333

Let a ♥ b be the greatest prime divisor of a + b.

At first, notice that from the initial 16 numbers we obtain 8 primes. The largest prime that can be

obtained is 31 = 15 ♥ 16; if this number occurs, the second largest can be 23 = 11 ♥ 12. Otherwise, 29

may occurs twice, from 16 ♥ 13 and 15 ♥ 14, followed by 19 – or lower.

From the stage when we are left with 8 primes, and after pairing them we get 4 primes. If a prime is

obtained from two odd primes a and b, then a ♥ b ≤ 2

ba+.

If else, at least one is 2 and let p be the other. The number p + 2 is prime only if p ∈ {3, 11, 17,

29}. Therefore, if p and q are prime with p ≤ q, then p ♥ g ≤ q + 2.

We will prove that the largest number which can end the game is 19. One example to obtain it is

exhibit below:

(1, 8); (2, 7); (9, 16); (10, 15); (3, 14); (4, 13); (5, 12); (6, 11) → 3, 3, 5, 5, 17, 17, 17, 17

(3, 3); (5, 5); (17, 17); (17, 17) → 3, 5, 17, 17

(3, 5); (17, 17) → 2, 17

(2, 17) → 19.

Now, we have to show that the game cannot end with a number strictly greater than 19. Since from

the second stage the number cannot increase with more than 2, and since 31 ♥ 2 = 11, we derive that

the game will end with a prime p ≤ 31. Suppose by contradiction that p ∈ {23, 29, 31}.

If p = 29, as 29 is not sum of two primes, then p is obtained from two of 29. In the previous stage

four 29’s are needed, then in the second stage eight 29’s are required, in contradiction with an initial

observation. Moreover, we have obtained a stronger result: 29 cannot end the game and cannot occur

even in the last pair, since after 2 steps at most one 29 may occur.

Suppose that p = 31. Two cases are possible: 31 = 2 ♥ 29 or 31 = 31 ♥ 31. The latter result forbids

Page 85: Carte Olimpiada_balcanica-2010

85858585

the first case, while the second case requires that the last four numbers are 31, 31, 31, 31 or 31, 31, 29,

2. But among the 8 primes obtained after the first step we have at most two 29’s or one 31, not enough

to produce three 31’s or two 31’s and one 29.

Assume that p = 23. Again two cases are possible: 23 = 29 ♥ 17 or 23 = 23 ♥ 23. The first case is

impossible as shown above, while the second case is allowed if the last four primes are 23, 23, 23, 23

or 29, 17, 23, 23. If all primes are 23, the previous step has eight numbers with the average of 23,

which is a contradiction with

8 ⋅ 23 < 1 + 2 + 3 + … + 16 = 8 ⋅ 17.

The second case lead similarly to contradiction, since 29 requires two 29’s and the pair of 23’s are

given by four numbers with the sum 4 ⋅ 23:

2 ⋅ 29 + 4 ⋅ 23 = 150 < 1 + 2 + … + 16 = 136.

The solution is now complete.

Solution to Solution to Solution to Solution to Problem Problem Problem Problem 111144444444

Consider an arbitrary grouping in pairs. A pair in which the persons cannot speak will be called

“bad”. If there are bad pairs, we prove that some changes can be made to decrease to number of bad

pairs. Applying this at most 4 times exhibit a grouping with no bad pairs, and we are done.

Label the pairs A, B, C, D and the persons in pair X by X1 and X2. Two persons that cannot converse

are called “enemies”, otherwise “friends”. Assume that A is a bad pair. Beside A1, the person A2 has at

most two other enemies. Two cases arise:

a) If the other enemies of A2 belong to the same pair – call it B, then A1 has at least a friend among

C1, C2, D1, D2.

Choose C1 as a friend of A1 and swap A1 with C2. The new pairs A and C are good, and the claim is

satisfied.

b) If not, in at least one of the pairs B, C, D there are only friends of A1. Wlog, say that this pair is

B. One of the persons in this pair must be a friend of A2; call this person B1. Now swap A1 with B1 and

the new pairs A and B are good, as desired.

Remark. Consider the graph with vertices in the eight persons and edges corresponding to each pair

of friends. The degree of each vertex is at least 4, so, according to Dirac’s theorem there exists a

hamiltonian cycle. Taking 4 edges with ho common endpoint from this cycle, we get 4 good pairs, as

needed.

Solution to Solution to Solution to Solution to Problem Problem Problem Problem 145145145145

Given a set X of n points in the plane, consider a maximal free subset Y made of m elements, hence

such that any point in X \ Y completes an equilateral triangle with (at least) a pair of points from Y.

(Any X contains free subsets, since any subset with 1 or 2 elements is obviously free.) But for any pair

of points from Y there exist only two points in the plane which complete an equilateral triangle, so

n – m ≤

22m

, that is n ≤ m2, or m ≥ n .

One checks the validity of this result for small values (1, 2, 3) of n, too (while the coplanarity

restriction is obvious).

Page 86: Carte Olimpiada_balcanica-2010

86868686

Solution to Solution to Solution to Solution to Problem Problem Problem Problem 111146464646

Label the rows from 1 to 8 and the columns from 1 to 8. The unit square which lies on the row i

and the column j will be referred as (i, j).

On the skew-diagonal {(i, i) | i = 1, 2, …, 8} there are exactly 2 squares in which checkers were

placed; wlog, assume that the squares are (1, 1), (2, 2). Looking at the 6 × 6 sub-array Q determined by

the rows 3-8 and the columns 3-8, we see that any “skew-diagonal” of Q, togheter with (1, 1), (2, 2), is

a skew-diagonal of the initial array. In view of the given conditions, no checkers are placed in the

squares of Q. Now take any skew-diagonal of Q with the squares (2, 1), (1, 2); this is a skew-diagonal

of the initial array, and the two checkers are placed inside (2, 1), (1, 2).

Up to this point, we know that checkers are placed in the squares on the rows 1-2 or on the columns

1-2. Suppose by way of contradiction that there exist a square located on the first two rows – say

(i, m), i = 1, 2, m ≥ 3 – and a square on the first two columns – say (s, j), j = 1, 2, s ≥ 3 – that hold

checkers. Then squares (i, m), (s, j), (3 – i, 3 – j) belongs to a skew-diagonal, contradiction.

Solution to Solution to Solution to Solution to Problem Problem Problem Problem 111147474747

For n ≥ 3, it suffices to draw 2n – 2 squares, as below:

• if n is odd, from each vertex V of P draw squares of sides 2

1−n squares n – 1, n – 2, …,

2

1+n,

with V as vertex in each square.

• if n is even, from each vertex V of P draw 2

n – 1 squares of sides n – 1, n – 2,

2

n + 1, with V as

vertex in each square and add 2 more squares of side 2

n with vertices in 2 opposite vertices of P.

To show that 2n – 2 square are needed, divide all four sides in unit segments with 4n – 4 points –

other than the vertices of the square – and consider for each point the unit segments perpendicular to

the border of the square. This 4n – 4 segments can be covered by no less than 2n – 2 squares, since a

square cannot cover 3 such segments, so we are done.

Solution to Solution to Solution to Solution to Problem Problem Problem Problem 141414148888

Two types of triangles arise after such a partition: “<” and “=”. Suppose that the initial triangle

ABC is of type < and notice that all three adjacent triangles – the ones sharing a common side – are of

type =. Call A-stripe the portion of plane containing the triangle ABC and bounded by the line BC and

the parallel from A to BC. (It is easy to observe that the plane is divided in stripes parallel to this

A-stripe.) Two =-triangles adjacent to ABC lie in this A-stripe. We claim that no path from ABC to

one of these triangles exists.

Indeed, observe that a move changes the type of the triangle hosting the checker, implying that if a

path must exists, then it has an odd number of moves. On the other hand, a move changes the position

of the checker from a stripe to an adjacent stripe located upwards or downwards. To reach in the end

the same A-stripe, such a path must consists in an even number of moves, a contradiction.

To conclude the proof, consider the B-stripe and the two =-triangles adjacent to ABC. Repeating

the argument, we are done.

Alternative Solution. Use four colors – one for ABC and the three adjacent triangles – to indicate

the triangles that can be visited starting from any of these triangles. Justify.

Page 87: Carte Olimpiada_balcanica-2010

87878787

Solution to Solution to Solution to Solution to Problem Problem Problem Problem 111149494949

Suppose A = a0, a1, …, 12 −n

a is a rearrangement of the numbers 0, 1, 2, …, 2n – 1 which satisfies

the required condition. Then (2A), (2A + 1) is a rearrangement of the numbers 0, 1, 2, …, 2n+1

– 1,

where (2A) = 2a0, 2a1, …, 212 −n

a and (2A + 1) = 2a0 + 1, 2a1 +1, …, 212 −n

a + 1. It is obvious that the

rearrangement (2A), (2A + 1) satisfies the claim. Indeed, no triples bi, bj, bk, with bi – bj = bj – bk,

i < j < k may occur in (2A) nor in (2A + 1), since either 2

,2

,2

kji bbb or

2

1,

2

1,

2

1 −−− kji bbb belongs to A,

contrary to the fact that A is “free” of triples in arithmetic progressions.

Starting with the arrangement 2, 0, 3, 1 for the numbers 0, 1, 2, 3 and applying the above

procedure, in 5 steps one has the required rearrangement.

Solution to Solution to Solution to Solution to Problem Problem Problem Problem 111150505050

The required number is 2006, the number of the subsets having 2005 elements.

For start, notice that each subset must have at least 2004 elements. If there exist a set with exactly

2004 elements, then this is unique and moreover, only 2 other subsets may be chosen.

If no set has 2004 elements, then we can choose among the 2006 subsets, with 2005 elements and

the set A with 2006 elements. But if A is among the chosen subsets, then any intersection will have

more than 2004 elements, false. The claim is now justified.

Solution to Solution to Solution to Solution to Problem Problem Problem Problem 151151151151

We claim v(m, n) = mn + 1.

Partition M into n sets Pk = {(x, y); n | x + y – k}, k = 1, 2, …, n. The pigeonhole principle now

forces (at least) m + 1 elements from P, be them Ai = (xi, yi), to belong to a same Pk. Now, if we

assume xi = xj, then from xi + yi – k ≡ xi + yi – k(mod n) it follows n | yi – yj, but as yi, yj ∈ {1, 2, …, n},

it follows yi = yj, i.e. Ai = Aj.

Conversely, mn + 1 is the least cardinality of P to warrant the claimed result; for |P| = mn, one can

pick P = {{x, y); 1 ≤ x ≤ m, 1 ≤ y ≤ n}; then any m + 1 elements from P, be them Ai = (xi, yi), will share

at least one xi = xj (pigeonhole principle again).

Solution to Solution to Solution to Solution to Problem Problem Problem Problem 152152152152

Consider all positive differences a – b among all 10 numbers. Since there are 210C = 45 positive

differences and all belong to the set 1, 2, 3, …, 36, at least two of them are equal. Let them be a – b

and c – d, with a > c. If a, b, c, d are all distinct, we are done; if not, then b = c, so b = c is one of the

8 numbers which are neither the lowest nor the greatest number from the initial ones.

Now observe that we have 45 positive differences and 36 possible values for them, so either 3

positive differences are equal or there are 9 pairs of equal positive differences.

The first case, gives a – b = c – d = e – f, with a > c > e. Since we cannot have b = c, b = e and d =

e, we are done.

The second case gives at least one pair of positive differences in which case b = c is excluded, as

only 8 candidates for b = c exist, so we are done.

Page 88: Carte Olimpiada_balcanica-2010

88888888

Solution to Solution to Solution to Solution to Problem Problem Problem Problem 151515153333

Consider a set S which satisfies all requirements. For each i ∈ A = {1, 2, …, n}, define Bi ⊂ B the

set of all elements j ∈ B for which the pair belongs to the set S – notice that some subsets Bi can be

empty. Counting all pairs in S over all second element in each pair, we have |S| = |B1| + |B2| + … + |Bn|.

The main idea is to observe the chain of ‘inequalities’

B1 ≤ B2 ≤ … ≤ Bn, where by X ≤ Y we mean that x ≤ y, for any x ∈ X and y ∈ Y, X, Y being sets of integers. (This

definition allows the empty set to occupy any position in this chain.)

Since B1 ∩ B2 ∩ … ∩ Bn = B = {1, 2, …, m} and any two consecutive subsets Bi share in common

at most one element, we get – by sieve theorem – that |S| ≤ m + n – 1, as claimed.

Solution to Solution to Solution to Solution to Problem Problem Problem Problem 151515154444

Divide the interior of the circle into 12 congruent sectors such that each marked point lies in the

interior of some sector. If one of them contains exactly 100 marked points, we are done. If not, we can

find a sector A containing less than 100 points and a sector B containing more than 100 points (observe

that it is not possible that all sectors contain less than 100 points or more than 100 points).

Rotate sector A towards sector B. At each moment at most one marked point gets in or out sector A.

It follows that there exists a moment in which the rotating sector contains exactly 100 marked points.

Solution to Solution to Solution to Solution to Problem Problem Problem Problem 155155155155

Let A1, A2, …, An be the vertices of the polygon. We start with the following

Lemma: Each segment AiAj belongs to at most 2 triangle of area 1 located on the same side of the

line AiAj.

Proof of the lemma. Indeed, suppose that on the same side of the line AiAj exist the vertices Am, An,

Ap, so that the triangles AiAjAm, AiAjAn and AiAjAp have the area 1. Then the points Am, An, Ap will be at

the same distance to the line AiAj, hence colinear. This is a contradiction, since the polygon is convex.

Consider first the n sides of the polygon. Each of them can form at most 2 triangles of area 1, as all

the vertices lie on the same side, hence we have by now at most 2n such triangles.

Consider now the n diagonals AiAi+2 – with the cyclic notations: An+j = Aj. Each of them can form at

most 3 triangles of area 1, one with Ai+1 and two with the vertices lying on the other side. Thus we

have at most 5n = 2n + 3n triangles.

Finally, consider the other diagonals of the polygon. They are 2

)5( −nn, and each of them can form

at most 4 triangles. The final counting is 5n + 42

)5( −nn = n(2n – 5), except that we have counted each

triangle three times, one time for each side. Therefore, there are at most 2

)52( −nn triangles, as

claimed.

Page 89: Carte Olimpiada_balcanica-2010

Table of Contents

FOREWORD ............................................................................................................................. 5

PROBLEMS

CHAPTER I. ALGEBRA .......................................................................................................................... 9

CHAPTER II. GEOMETRY .................................................................................................................... 16

CHAPTER III. NUMBER THEORY ........................................................................................................ 24

CHAPTER IV. COMBINATORICS ......................................................................................................... 29

FORMAL SOLUTIONS35

CHAPTER I. ALGEBRA ....................................................................................................................... 37

CHAPTER II. GEOMETRY .................................................................................................................. 49

CHAPTER III. NUMBER THEORY ...................................................................................................... 67

CHAPTER IV. COMBINATORICS .............................................................................................. 79

Page 90: Carte Olimpiada_balcanica-2010

Tiparul executat la Tipografia Editurii Paralela 45

COMENZI – CARTEA PRIN POŞTĂ EDITURA PARALELA 45 Piteşti, jud. Argeş, cod 110174, str. FraŃii Goleşti 130 Tel./fax: 0248 214 533; 0248 631 439; 0248 631 492 Tel.: 0753 040 444;

0721 247 918. E-mail: [email protected] sau accesaŃi www.edituraparalela45.ro